*NURSING > EXAM > NUTRITION EXAM 2: STUDY GUIDE CHAPTER 1_Latest,100% CORRECT (All)

NUTRITION EXAM 2: STUDY GUIDE CHAPTER 1_Latest,100% CORRECT

Document Content and Description Below

After completing an initial assessment of a patient, the nurse has charted that his respirations are eupneic and his pulse is 58 beats per minute. These types of data would be: a. Objective. b. Refl... ective. c. Subjective. d. Introspective. A Objective data are what the health professional observes by inspecting, percussing, palpating, and auscultating during the physical examination. Subjective data is what the person says about him or herself during history taking. The terms reflective and introspective are not used to describe data. 2. A patient tells the nurse that he is very nervous, is nauseated, and “feels hot.” These types of data would be: a. Objective. b. Reflective. c. Subjective. d. Introspective. C Subjective data are what the person says about him or herself during history taking. Objective data are what the health professional observes by inspecting, percussing, palpating, and auscultating during the physical examination. The terms reflective and introspective are not used to describe data. 3. The patient’s record, laboratory studies, objective data, and subjective data combine to form the: a. Data base. b. Admitting data. c. Financial statement. d. Discharge summary. A Together with the patient’s record and laboratory studies, the objective and subjective data form the data base. The other items are not part of the patient’s record, laboratory studies, or data. 4. When listening to a patient’s breath sounds, the nurse is unsure of a sound that is heard. The nurse’s next action should be to: a. Immediately notify the patient’s physician. b. Document the sound exactly as it was heard. c. Validate the data by asking a coworker to listen to the breath sounds. d. Assess again in 20 minutes to note whether the sound is still present. C When unsure of a sound heard while listening to a patient’s breath sounds, the nurse validates the data to ensure accuracy. If the nurse has less experience in an area, then he or she asks an expert to listen. 5. The nurse is conducting a class for new graduate nurses. During the teaching session, the nurse should keep in mind that novice nurses, without a background of skills and experience from which to draw, are more likely to make their decisions using: a. Intuition. b. A set of rules. c. Articles in journals. d. Advice from supervisors. B Novice nurses operate from a set of defined, structured rules. The expert practitioner uses intuitive links. 6. Expert nurses learn to attend to a pattern of assessment data and act without consciously labeling it. These responses are referred to as: a. Intuition. b. The nursing process. c. Clinical knowledge. d. Diagnostic reasoning. A Intuition is characterized by pattern recognition—expert nurses learn to attend to a pattern of assessment data and act without consciously labeling it. The other options are not correct. 7. The nurse is reviewing information about evidence-based practice (EBP). Which statement best reflects EBP? a. EBP relies on tradition for support of best practices. b. EBP is simply the use of best practice techniques for the treatment of patients. c. EBP emphasizes the use of best evidence with the clinician’s experience. d. The patient’s own preferences are not important with EBP. C EBP is a systematic approach to practice that emphasizes the use of best evidence in combination with the clinician’s experience, as well as patient preferences and values, when making decisions about care and treatment. EBP is more than simply using the best practice techniques to treat patients, and questioning tradition is important when no compelling and supportive research evidence exists. 8. The nurse is conducting a class on priority setting for a group of new graduate nurses. Which is an example of a first-level priority problem? a. Patient with postoperative pain b. Newly diagnosed patient with diabetes who needs diabetic teaching c. Individual with a small laceration on the sole of the foot d. Individual with shortness of breath and respiratory distress D First-level priority problems are those that are emergent, life threatening, and immediate (e.g., establishing an airway, supporting breathing, maintaining circulation, monitoring abnormal vital signs) (see Table 1-1). 9. When considering priority setting of problems, the nurse keeps in mind that second-level priority problems include which of these aspects? a. Low self-esteem b. Lack of knowledge c. Abnormal laboratory values d. Severely abnormal vital signs C Second-level priority problems are those that require prompt intervention to forestall further deterioration (e.g., mental status change, acute pain, abnormal laboratory values, risks to safety or security) (see Table 1-1). 10. Which critical thinking skill helps the nurse see relationships among the data? a. Validation b. Clustering related cues c. Identifying gaps in data d. Distinguishing relevant from irrelevant B Clustering related cues helps the nurse see relationships among the data. 11. The nurse knows that developing appropriate nursing interventions for a patient relies on the appropriateness of the __________ diagnosis. a. Nursing b. Medical c. Admission d. Collaborative A An accurate nursing diagnosis provides the basis for the selection of nursing interventions to achieve outcomes for which the nurse is accountable. The other items do not contribute to the development of appropriate nursing interventions. 12. The nursing process is a sequential method of problem solving that nurses use and includes which steps? a. Assessment, treatment, planning, evaluation, discharge, and follow-up b. Admission, assessment, diagnosis, treatment, and discharge planning c. Admission, diagnosis, treatment, evaluation, and discharge planning d. Assessment, diagnosis, outcome identification, planning, implementation, and evaluation D The nursing process is a method of problem solving that includes assessment, diagnosis, outcome identification, planning, implementation, and evaluation. 13. A newly admitted patient is in acute pain, has not been sleeping well lately, and is having difficulty breathing. How should the nurse prioritize these problems? a. Breathing, pain, and sleep b. Breathing, sleep, and pain c. Sleep, breathing, and pain d. Sleep, pain, and breathing A First-level priority problems are immediate priorities, remembering the ABCs (airway, breathing, and circulation), followed by second-level problems, and then third-level problems. 14. Which of these would be formulated by a nurse using diagnostic reasoning? a. Nursing diagnosis b. Medical diagnosis c. Diagnostic hypothesis d. Diagnostic assessment C Diagnostic reasoning calls for the nurse to formulate a diagnostic hypothesis; the nursing process calls for a nursing diagnosis. 15. Barriers to incorporating EBP include: a. Nurses’ lack of research skills in evaluating the quality of research studies. b. Lack of significant research studies. c. Insufficient clinical skills of nurses. d. Inadequate physical assessment skills. A As individuals, nurses lack research skills in evaluating the quality of research studies, are isolated from other colleagues who are knowledgeable in research, and often lack the time to visit the library to read research. The other responses are not considered barriers. 16. What step of the nursing process includes data collection by health history, physical examination, and interview? a. Planning b. Diagnosis c. Evaluation d. Assessment D Data collection, including performing the health history, physical examination, and interview, is the assessment step of the nursing process (see Figure 1-2). 17. During a staff meeting, nurses discuss the problems with accessing research studies to incorporate evidence-based clinical decision making into their practice. Which suggestion by the nurse manager would best help these problems? a. Form a committee to conduct research studies. b. Post published research studies on the unit’s bulletin boards. c. Encourage the nurses to visit the library to review studies. d. Teach the nurses how to conduct electronic searches for research studies. D Facilitating support for EBP would include teaching the nurses how to conduct electronic searches; time to visit the library may not be available for many nurses. Actually conducting research studies may be helpful in the long-run but not an immediate solution to reviewing existing research. 18. When reviewing the concepts of health, the nurse recalls that the components of holistic health include which of these? a. Disease originates from the external environment. b. The individual human is a closed system. c. Nurses are responsible for a patient’s health state. d. Holistic health views the mind, body, and spirit as interdependent. D Consideration of the whole person is the essence of holistic health, which views the mind, body, and spirit as interdependent. The basis of disease originates from both the external environment and from within the person. Both the individual human and the external environment are open systems, continually changing and adapting, and each person is responsible for his or her own personal health state. 19. The nurse recognizes that the concept of prevention in describing health is essential because: a. Disease can be prevented by treating the external environment. b. The majority of deaths among Americans under age 65 years are not preventable. c. Prevention places the emphasis on the link between health and personal behavior. d. The means to prevention is through treatment provided by primary health care practitioners. C A natural progression to prevention rounds out the present concept of health. Guidelines to prevention place the emphasis on the link between health and personal behavior. 20. The nurse is performing a physical assessment on a newly admitted patient. An example of objective information obtained during the physical assessment includes the: a. Patient’s history of allergies. b. Patient’s use of medications at home. c. Last menstrual period 1 month ago. d. 2  5 cm scar on the right lower forearm. D Objective data are the patient’s record, laboratory studies, and condition that the health professional observes by inspecting, percussing, palpating, and auscultating during the physical examination. The other responses reflect subjective data. 21. A visiting nurse is making an initial home visit for a patient who has many chronic medical problems. Which type of data base is most appropriate to collect in this setting? a. A follow-up data base to evaluate changes at appropriate intervals b. An episodic data base because of the continuing, complex medical problems of this patient c. A complete health data base because of the nurse’s primary responsibility for monitoring the patient’s health d. An emergency data base because of the need to collect information and make accurate diagnoses rapidly C The complete data base is collected in a primary care setting, such as a pediatric or family practice clinic, independent or group private practice, college health service, women’s health care agency, visiting nurse agency, or community health agency. In these settings, the nurse is the first health professional to see the patient and has the primary responsibility for monitoring the person’s health care. 22. Which situation is most appropriate during which the nurse performs a focused or problem-centered history? a. Patient is admitted to a long-term care facility. b. Patient has a sudden and severe shortness of breath. c. Patient is admitted to the hospital for surgery the following day. d. Patient in an outpatient clinic has cold and influenza-like symptoms. D In a focused or problem-centered data base, the nurse collects a “mini” data base, which is smaller in scope than the completed data base. This mini data base primarily concerns one problem, one cue complex, or one body system. 23. A patient is at the clinic to have her blood pressure checked. She has been coming to the clinic weekly since she changed medications 2 months ago. The nurse should: a. Collect a follow-up data base and then check her blood pressure. b. Ask her to read her health record and indicate any changes since her last visit. c. Check only her blood pressure because her complete health history was documented 2 months ago. d. Obtain a complete health history before checking her blood pressure because much of her history information may have changed. A A follow-up data base is used in all settings to follow up short-term or chronic health problems. The other responses are not appropriate for the situation. 24. A patient is brought by ambulance to the emergency department with multiple traumas received in an automobile accident. He is alert and cooperative, but his injuries are quite severe. How would the nurse proceed with data collection? a. Collect history information first, then perform the physical examination and institute life-saving measures. b. Simultaneously ask history questions while performing the examination and initiating life-saving measures. c. Collect all information on the history form, including social support patterns, strengths, and coping patterns. d. Perform life-saving measures and delay asking any history questions until the patient is transferred to the intensive care unit. B The emergency data base calls for a rapid collection of the data base, often concurrently compiled with life-saving measures. The other responses are not appropriate for the situation. 25. A 42-year-old patient of Asian descent is being seen at the clinic for an initial examination. The nurse knows that including cultural information in his health assessment is important to: a. Identify the cause of his illness. b. Make accurate disease diagnoses. c. Provide cultural health rights for the individual. d. Provide culturally sensitive and appropriate care. D The inclusion of cultural considerations in the health assessment is of paramount importance to gathering data that are accurate and meaningful and to intervening with culturally sensitive and appropriate care. 26. In the health promotion model, the focus of the health professional includes: a. Changing the patient’s perceptions of disease. b. Identifying biomedical model interventions. c. Identifying negative health acts of the consumer. d. Helping the consumer choose a healthier lifestyle. D In the health promotion model, the focus of the health professional is on helping the consumer choose a healthier lifestyle. 27. The nurse has implemented several planned interventions to address the nursing diagnosis of acute pain. Which would be the next appropriate action? a. Establish priorities. b. Identify expected outcomes. c. Evaluate the individual’s condition, and compare actual outcomes with expected outcomes. d. Interpret data, and then identify clusters of cues and make inferences. C Evaluation is the next step after the implementation phase of the nursing process. During this step, the nurse evaluates the individual’s condition and compares the actual outcomes with expected outcomes (See Figure 1-2). 28. Which statement best describes a proficient nurse? A proficient nurse is one who: a. Has little experience with a specified population and uses rules to guide performance. b. Has an intuitive grasp of a clinical situation and quickly identifies the accurate solution. c. Sees actions in the context of daily plans for patients. d. Understands a patient situation as a whole rather than a list of tasks and recognizes the long-term goals for the patient. D The proficient nurse, with more time and experience than the novice nurse, is able to understand a patient situation as a whole rather than as a list of tasks. The proficient nurse is able to see how today’s nursing actions can apply to the point the nurse wants the patient to reach at a future time. MULTIPLE RESPONSE 1. The nurse is reviewing data collected after an assessment. Of the data listed below, which would be considered related cues that would be clustered together during data analysis? Select all that apply. a. Inspiratory wheezes noted in left lower lobes b. Hypoactive bowel sounds c. Nonproductive cough d. Edema, +2, noted on left hand e. Patient reports dyspnea upon exertion f. Rate of respirations 16 breaths per minute A, C, E, F Clustering related cues help the nurse recognize relationships among the data. The cues related to the patient’s respiratory status (e.g., wheezes, cough, report of dyspnea, respiration rate and rhythm) are all related. Cues related to bowels and peripheral edema are not related to the respiratory cues. MATCHING Put the following patient situations in order according to the level of priority. a. A patient newly diagnosed with type 2 diabetes mellitus does not know how to check his own blood glucose levels with a glucometer. b. A teenager who was stung by a bee during a soccer match is having trouble breathing. c. An older adult with a urinary tract infection is also showing signs of confusion and agitation. 1. a = First-level priority problem 2. b = Second-level priority problem 3. c = Third-level priority problem 1. B 2. C 3. A CHAPTER 6 1. A woman has come to the clinic to seek help with a substance abuse problem. She admits to using cocaine just before arriving. Which of these assessment findings would the nurse expect to find when examining this woman? a. Dilated pupils, pacing, and psychomotor agitation b. Dilated pupils, unsteady gait, and aggressiveness c. Pupil constriction, lethargy, apathy, and dysphoria d. Constricted pupils, euphoria, and decreased temperature A A cocaine user’s appearance includes pupillary dilation, tachycardia or bradycardia, elevated or lowered blood pressure, sweating, chills, nausea, vomiting, and weight loss. The person’s behavior includes euphoria, talkativeness, hypervigilance, pacing, psychomotor agitation, impaired social or occupational functioning, fighting, grandiosity, and visual or tactile hallucinations. 2. The nurse is assessing a patient who has been admitted for cirrhosis of the liver, secondary to chronic alcohol use. During the physical assessment, the nurse looks for cardiac problems that are associated with chronic use of alcohol, such as: a. Hypertension. b. Ventricular fibrillation. c. Bradycardia. d. Mitral valve prolapse. A Even moderate drinking leads to hypertension and cardiomyopathy, with an increase in left ventricular mass, dilation of ventricles, and wall thinning. Ventricular fibrillation, bradycardia, and mitral valve prolapse are not associated with chronic heavy use of alcohol. 3. The nurse is conducting a class on alcohol and the effects of alcohol on the body. How many standard drinks (each containing 14 grams of alcohol) per day in men are associated with increased deaths from cirrhosis, cancers of the mouth, esophagus, and injuries? a. 2 b. 4 c. 6 d. 8 B In men, alcohol consumption of at least four standard drinks per day is associated with increased deaths from liver cirrhosis, cancers of the mouth, esophagus and other areas, and deaths from injuries and other external causes. 4. During a session on substance abuse, the nurse is reviewing statistics with the class. For persons aged 12 years and older, which illicit substance was most commonly used? a. Crack cocaine b. Heroin c. Marijuana d. Hallucinogens C In persons age 12 years and older who reported using during the past month, marijuana (hashish) was the most commonly used illicit drug reported. 5. A woman who has just discovered that she is pregnant is in the clinic for her first obstetric visit. She asks the nurse, “How many drinks a day is safe for my baby?” The nurse’s best response is: a. “You should limit your drinking to once or twice a week.” b. “It’s okay to have up to two glasses of wine a day.” c. “As long as you avoid getting drunk, you should be safe.” d. “No amount of alcohol has been determined to be safe during pregnancy.” D No amount of alcohol has been determined to be safe for pregnant women. The potential adverse effects of alcohol use on the fetus are well known; women who are pregnant should be screened for alcohol use, and abstinence should be recommended. 6. When reviewing the use of alcohol by older adults, the nurse notes that older adults have several characteristics that can increase the risk of alcohol use. Which would increase the bioavailability of alcohol in the blood for longer periods in the older adult? a. Increased muscle mass b. Decreased liver and kidney functioning c. Decreased blood pressure d. Increased cardiac output B Decreased liver and kidney functioning increases the bioavailability of alcohol in the blood for longer periods. Aging people experience decreased muscle mass (not increased), which also increases the alcohol concentration in the blood because the alcohol is distributed to less tissue over time. Blood pressure and cardiac output are not factors regarding bioavailability. 7. During an assessment, the nurse asks a female patient, “How many alcoholic drinks do you have a week?” Which answer by the patient would indicate at-risk drinking? a. “I may have one or two drinks a week.” b. “I usually have three or four drinks a week.” c. “I’ll have a glass or two of wine every now and then.” d. “I have seven or eight drinks a week, but I never get drunk.” D For women, having seven or more drinks a week or three or more drinks per occasion is considered at-risk drinking, according to the National Institute on Alcohol Abuse and Alcoholism. 8. The nurse is asking an adolescent about illicit substance abuse. The adolescent answers, “Yes, I’ve used marijuana at parties with my friends.” What is the next question the nurse should ask? a. “Who are these friends?” b. “Do your parents know about this?” c. “When was the last time you used marijuana?” d. “Is this a regular habit?” C If a patient admits to the use of illicit substances, then the nurse should ask, “When was the last time you used drugs?” and “How much did you take that time?” The other questions may be considered accusatory and are not conducive to gathering information. 9. The nurse has completed an assessment on a patient who came to the clinic for a leg injury. As a result of the assessment, the nurse has determined that the patient has at-risk alcohol use. Which action by the nurse is most appropriate at this time? a. Record the results of the assessment, and notify the physician on call. b. State, “You are drinking more than is medically safe. I strongly recommend that you quit drinking, and I’m willing to help you.” c. State, “It appears that you may have a drinking problem. Here is the telephone number of our local Alcoholics Anonymous chapter.” d. Give the patient information about a local rehabilitation clinic. B If an assessment has determined that the patient has at-risk drinking behavior, then the nurse should give a short but clear statement of assistance and concern. Simply giving out a telephone number or referral to agencies may not be enough. 10. A patient is brought to the emergency department. He is restless, has dilated pupils, is sweating, has a runny nose and tearing eyes, and complains of muscle and joint pains. His girlfriend thinks he has influenza, but she became concerned when his temperature went up to 39.4° C. She admits that he has been a heavy drug user, but he has been trying to stop on his own. The nurse suspects that the patient is experiencing withdrawal symptoms from which substance? a. Alcohol b. Heroin c. Crack cocaine d. Sedatives B Withdrawal symptoms of opiates, such as heroin, are similar to the clinical picture of influenza and include symptoms such as dilated pupils, lacrimation, runny nose, tachycardia, fever, restlessness, muscle and joint pains, and other symptoms. (Withdrawal symptoms from alcohol, cocaine, and sedatives are described in Table 6-7.) 11. The nurse is reviewing aspects of substance abuse in preparation for a seminar. Which of these statements illustrates the concept of tolerance to an illicit substance? The person: a. Has a physiologic dependence on a substance. b. Requires an increased amount of the substance to produce the same effect. c. Requires daily use of the substance to function and is unable to stop using it. d. Experiences a syndrome of physiologic symptoms if the substance is not used. B The concept of tolerance to a substance indicates that the person requires an increased amount of the substance to produce the same effect. Abuse indicates that the person needs to use the substance daily to function, and the person is unable to stop using it. Dependence is an actual physiologic dependence on the substance. Withdrawal occurs when cessation of the substance produces a syndrome of physiologic symptoms. MULTIPLE RESPONSE 1. A patient with a known history of heavy alcohol use has been admitted to the ICU after he was found unconscious outside a bar. The nurse closely monitors him for symptoms of withdrawal. Which of these symptoms may occur during this time? Select all that apply. a. Bradycardia b. Coarse tremor of the hands c. Transient hallucinations d. Somnolence e. Sweating B, C, E Symptoms of uncomplicated alcohol withdrawal start shortly after the cessation of drinking, peak at the second day, and improve by the fourth or fifth day. Symptoms include coarse tremors of the hands, tongue, and eyelids; anorexia; nausea and vomiting; autonomic hyperactivity (e.g., tachycardia, sweating, elevated blood pressure); and transient hallucinations, among other symptoms (see Table 6-7). 2. A patient visits the clinic to ask about smoking cessation. He has smoked heavily for 30 years and wants to stop “cold turkey.” He asks the nurse, “What symptoms can I expect if I do this?” Which of these symptoms should the nurse share with the patient as possible symptoms of nicotine withdrawal? Select all that apply. a. Headaches b. Hunger c. Sleepiness d. Restlessness e. Nervousness f. Sweating A, B, D, E Symptoms of nicotine withdrawal include vasodilation, headaches, anger, irritability, frustration, anxiety, nervousness, awakening at night, difficulty concentrating, depression, hunger, impatience, and the desire to smoke (see Table 6-7). CHAPTER 7 1. As a mandatory reporter of elder abuse, which must be present before a nurse should notify the authorities? a. Statements from the victim b. Statements from witnesses c. Proof of abuse and/or neglect d. Suspicion of elder abuse and/or neglect D Many health care workers are under the erroneous assumption that proof is required before notification of suspected abuse can occur. Only the suspicion of elder abuse or neglect is necessary. 2. During a home visit, the nurse notices that an older adult woman is caring for her bedridden husband. The woman states that this is her duty, she does the best she can, and her children come to help when they are in town. Her husband is unable to care for himself, and she appears thin, weak, and exhausted. The nurse notices that several of his prescription medication bottles are empty. This situation is best described by the term: a. Physical abuse. b. Financial neglect. c. Psychological abuse. d. Unintentional physical neglect. D Unintentional physical neglect may occur, despite good intentions, and is the failure of a family member or caregiver to provide basic goods or services. Physical abuse is defined as violent acts that result or could result in injury, pain, impairment, or disease. Financial neglect is defined as the failure to use the assets of the older person to provide services needed by him or her. Psychological abuse is defined as behaviors that result in mental anguish. 3. The nurse is aware that intimate partner violence (IPV) screening should occur with which situation? a. When IPV is suspected b. When a woman has an unexplained injury c. As a routine part of each health care encounter d. When a history of abuse in the family is known C Many nursing professional organizations have called for routine, universal screening for IPV to assist women in getting help for the problem. 4. Which statement is best for the nurse to use when preparing to administer the Abuse Assessment Screen? a. “We are required by law to ask these questions.” b. “We need to talk about whether you believe you have been abused.” c. “We are asking these questions because we suspect that you are being abused.” d. “We need to ask the following questions because domestic violence is so common in our society.” D Such an introduction alerts the woman that questions about domestic violence are coming and ensures the woman that she is not being singled out for these questions. 5. Which term refers to a wound produced by the tearing or splitting of body tissue, usually from blunt impact over a bony surface? a. Abrasion b. Contusion c. Laceration d. Hematoma C The term laceration refers to a wound produced by the tearing or splitting of body tissue. An abrasion is caused by the rubbing of the skin or mucous membrane. A contusion is injury to tissues without breakage of skin, and a hematoma is a localized collection of extravasated blood. 6. During an examination, the nurse notices a patterned injury on a patient’s back. Which of these would cause such an injury? a. Blunt force b. Friction abrasion c. Stabbing from a kitchen knife d. Whipping from an extension cord D A patterned injury is an injury caused by an object that leaves a distinct pattern on the skin or organ. The other actions do not cause a patterned injury. 7. When documenting IPV and elder abuse, the nurse should include: a. Photographic documentation of the injuries. b. Summary of the abused patient’s statements. c. Verbatim documentation of every statement made. d. General description of injuries in the progress notes. A Documentation of IPV and elder abuse must include detailed nonbiased progress notes, the use of injury maps, and photographic documentation. Written documentation needs to be verbatim, within reason. Not every statement can be documented. 8. A female patient has denied any abuse when answering the Abuse Assessment Screen, but the nurse has noticed some other conditions that are associated with IPV. Examples of such conditions include: a. Asthma. b. Confusion. c. Depression. d. Frequent colds. C Depression is one of the conditions that is particularly associated with IPV. Abused women also have been found to have more chronic health problems, such as neurologic, gastrointestinal, and gynecologic symptoms; chronic pain; and symptoms of suicidality and posttraumatic stress disorder. 9. The nurse is using the danger assessment (DA) tool to evaluate the risk of homicide. Which of these statements best describes its use? a. The DA tool is to be administered by law enforcement personnel. b. The DA tool should be used in every assessment of suspected abuse. c. The number of “yes” answers indicates the woman’s understanding of her situation. d. The higher the number of “yes” answers, the more serious the danger of the woman’s situation. D No predetermined cutoff scores exist on the DA. The higher the number “yes” answers, the more serious the danger of the woman’s situation. The use of this tool is not limited to law enforcement personnel and is not required in every case of suspected abuse. 10. The nurse is assessing bruising on an injured patient. Which color indicates a new bruise that is less than 2 hours old? a. Red b. Purple-blue c. Greenish-brown d. Brownish-yellow A A new bruise is usually red and will often develop a purple or purple-blue appearance 12 to 36 hours after blunt-force trauma. The color of bruises (and ecchymoses) generally progresses from purple-blue to bluish-green to greenish-brown to brownish-yellow before fading away. 11. The nurse suspects abuse when a 10-year-old child is taken to the urgent care center for a leg injury. The best way to document the history and physical findings is to: a. Document what the child’s caregiver tells the nurse. b. Use the words the child has said to describe how the injury occurred. c. Record what the nurse observes during the conversation. d. Rely on photographs of the injuries. B When documenting the history and physical findings of suspected child abuse and neglect, use the words the child has said to describe how his or her injury occurred. Remember, the abuser may be accompanying the child. 12. During an interview, a woman has answered “yes” to two of the Abuse Assessment Screen questions. What should the nurse say next? a. “I need to report this abuse to the authorities.” b. “Tell me about this abuse in your relationship.” c. “So you were abused?” d. “Do you know what caused this abuse?” B If a woman answers “yes” to any of the Abuse Assessment Screen questions, then the nurse should ask questions designed to assess how recent and how serious the abuse was. Asking the woman an open-ended question, such as “tell me about this abuse in your relationship” is a good way to start. 13. The nurse is examining a 3-year-old child who was brought to the emergency department after a fall. Which bruise, if found, would be of most concern? a. Bruise on the knee b. Bruise on the elbow c. Bruising on the abdomen d. Bruise on the shin C Studies have shown that children who are walking often have bruises over the bony prominences of the front of their bodies. Other studies have found that bruising in atypical places such as the buttocks, hands, feet, and abdomen were exceedingly rare and should arouse concern. MULTIPLE RESPONSE 1. The nurse assesses an older woman and suspects physical abuse. Which questions are appropriate for screening for abuse? Select all that apply. a. “Has anyone made you afraid, touched you in ways that you did not want, or hurt you physically?” b. “Are you being abused?” c. “Have you relied on people for any of the following: bathing, dressing, shopping, banking, or meals?” d. “Have you been upset because someone talked to you in a way that made you feel shamed or threatened?” e. “Have you relied on people for any of the following: bathing, dressing, shopping, banking, or meals?” A, C, D, E Directly asking “Are you being abused?” is not an appropriate screening question for abuse because the woman could easily say “no,” and no further information would be obtained. The other questions are among the questions recommended by the Elder Abuse Suspicion Index (EASI) when screening for elder abuse. CHAPER 8 1. When performing a physical assessment, the first technique the nurse will always use is: a. Palpation. b. Inspection. c. Percussion. d. Auscultation. B The skills requisite for the physical examination are inspection, palpation, percussion, and auscultation. The skills are performed one at a time and in this order (with the exception of the abdominal assessment, during which auscultation takes place before palpation and percussion). The assessment of each body system begins with inspection. A focused inspection takes time and yields a surprising amount of information. 2. The nurse is preparing to perform a physical assessment. Which statement is true about the physical assessment? The inspection phase: a. Usually yields little information. b. Takes time and reveals a surprising amount of information. c. May be somewhat uncomfortable for the expert practitioner. d. Requires a quick glance at the patient’s body systems before proceeding with palpation. B A focused inspection takes time and yields a surprising amount of information. Initially, the examiner may feel uncomfortable, staring at the person without also doing something. A focused assessment is significantly more than a “quick glance.” 3. The nurse is assessing a patient’s skin during an office visit. What part of the hand and technique should be used to best assess the patient’s skin temperature? a. Fingertips; they are more sensitive to small changes in temperature. b. Dorsal surface of the hand; the skin is thinner on this surface than on the palms. c. Ulnar portion of the hand; increased blood supply in this area enhances temperature sensitivity. d. Palmar surface of the hand; this surface is the most sensitive to temperature variations because of its increased nerve supply in this area. B The dorsa (backs) of the hands and fingers are best for determining temperature because the skin is thinner on the dorsal surfaces than on the palms. Fingertips are best for fine, tactile discrimination. The other responses are not useful for palpation. 4. Which of these techniques uses the sense of touch to assess texture, temperature, moisture, and swelling when the nurse is assessing a patient? a. Palpation b. Inspection c. Percussion d. Auscultation A Palpation uses the sense of touch to assess the patient for these factors. Inspection involves vision; percussion assesses through the use of palpable vibrations and audible sounds; and auscultation uses the sense of hearing. 5. The nurse is preparing to assess a patient’s abdomen by palpation. How should the nurse proceed? a. Palpation of reportedly “tender” areas are avoided because palpation in these areas may cause pain. b. Palpating a tender area is quickly performed to avoid any discomfort that the patient may experience. c. The assessment begins with deep palpation, while encouraging the patient to relax and to take deep breaths. d. The assessment begins with light palpation to detect surface characteristics and to accustom the patient to being touched. D Light palpation is initially performed to detect any surface characteristics and to accustom the person to being touched. Tender areas should be palpated last, not first. 6. The nurse would use bimanual palpation technique in which situation? a. Palpating the thorax of an infant b. Palpating the kidneys and uterus c. Assessing pulsations and vibrations d. Assessing the presence of tenderness and pain B Bimanual palpation requires the use of both hands to envelop or capture certain body parts or organs such as the kidneys, uterus, or adnexa. The other situations are not appropriate for bimanual palpation. 7. The nurse is preparing to percuss the abdomen of a patient. The purpose of the percussion is to assess the __________ of the underlying tissue. a. Turgor b. Texture c. Density d. Consistency C Percussion yields a sound that depicts the location, size, and density of the underlying organ. Turgor and texture are assessed with palpation. 8. The nurse is reviewing percussion techniques with a newly graduated nurse. Which technique, if used by the new nurse, indicates that more review is needed? a. Percussing once over each area b. Quickly lifting the striking finger after each stroke c. Striking with the fingertip, not the finger pad d. Using the wrist to make the strikes, not the arm A For percussion, the nurse should percuss two times over each location. The striking finger should be quickly lifted because a resting finger damps off vibrations. The tip of the striking finger should make contact, not the pad of the finger. The wrist must be relaxed and is used to make the strikes, not the arm. 9. When percussing over the liver of a patient, the nurse notices a dull sound. The nurse should: a. Consider this a normal finding. b. Palpate this area for an underlying mass. c. Reposition the hands, and attempt to percuss in this area again. d. Consider this finding as abnormal, and refer the patient for additional treatment. A Percussion over relatively dense organs, such as the liver or spleen, will produce a dull sound. The other responses are not correct. 10. The nurse is unable to identify any changes in sound when percussing over the abdomen of an obese patient. What should the nurse do next? a. Ask the patient to take deep breaths to relax the abdominal musculature. b. Consider this finding as normal, and proceed with the abdominal assessment. c. Increase the amount of strength used when attempting to percuss over the abdomen. d. Decrease the amount of strength used when attempting to percuss over the abdomen. C The thickness of the person’s body wall will be a factor. The nurse needs a stronger percussion stroke for persons with obese or very muscular body walls. The force of the blow determines the loudness of the note. The other actions are not correct. 11. The nurse hears bilateral loud, long, and low tones when percussing over the lungs of a 4-year-old child. The nurse should: a. Palpate over the area for increased pain and tenderness. b. Ask the child to take shallow breaths, and percuss over the area again. c. Immediately refer the child because of an increased amount of air in the lungs. d. Consider this finding as normal for a child this age, and proceed with the examination. D Percussion notes that are loud in amplitude, low in pitch, of a booming quality, and long in duration are normal over a child’s lung. 12. A patient has suddenly developed shortness of breath and appears to be in significant respiratory distress. After calling the physician and placing the patient on oxygen, which of these actions is the best for the nurse to take when further assessing the patient? a. Count the patient’s respirations. b. Bilaterally percuss the thorax, noting any differences in percussion tones. c. Call for a chest x-ray study, and wait for the results before beginning an assessment. d. Inspect the thorax for any new masses and bleeding associated with respirations. B Percussion is always available, portable, and offers instant feedback regarding changes in underlying tissue density, which may yield clues of the patient’s physical status. 13. The nurse is teaching a class on basic assessment skills. Which of these statements is true regarding the stethoscope and its use? a. Slope of the earpieces should point posteriorly (toward the occiput). b. Although the stethoscope does not magnify sound, it does block out extraneous room noise. c. Fit and quality of the stethoscope are not as important as its ability to magnify sound. d. Ideal tubing length should be 22 inches to dampen the distortion of sound. B The stethoscope does not magnify sound, but it does block out extraneous room sounds. The slope of the earpieces should point forward toward the examiner’s nose. Long tubing will distort sound. The fit and quality of the stethoscope are both important. 14. The nurse is preparing to use a stethoscope for auscultation. Which statement is true regarding the diaphragm of the stethoscope? The diaphragm: a. Is used to listen for high-pitched sounds. b. Is used to listen for low-pitched sounds. c. Should be lightly held against the person’s skin to block out low-pitched sounds. d. Should be lightly held against the person’s skin to listen for extra heart sounds and murmurs. A The diaphragm of the stethoscope is best for listening to high-pitched sounds such as breath, bowel, and normal heart sounds. It should be firmly held against the person’s skin, firmly enough to leave a ring. The bell of the stethoscope is best for soft, low-pitched sounds such as extra heart sounds or murmurs. 15. Before auscultating the abdomen for the presence of bowel sounds on a patient, the nurse should: a. Warm the endpiece of the stethoscope by placing it in warm water. b. Leave the gown on the patient to ensure that he or she does not get chilled during the examination. c. Ensure that the bell side of the stethoscope is turned to the “on” position. d. Check the temperature of the room, and offer blankets to the patient if he or she feels cold. D The examination room should be warm. If the patient shivers, then the involuntary muscle contractions can make it difficult to hear the underlying sounds. The end of the stethoscope should be warmed between the examiner’s hands, not with water. The nurse should never listen through a gown. The diaphragm of the stethoscope should be used to auscultate for bowel sounds. 16. The nurse will use which technique of assessment to determine the presence of crepitus, swelling, and pulsations? a. Palpation b. Inspection c. Percussion d. Auscultation A Palpation applies the sense of touch to assess texture, temperature, moisture, organ location and size, as well as any swelling, vibration or pulsation, rigidity or spasticity, crepitation, presence of lumps or masses, and the presence of tenderness or pain. 17. The nurse is preparing to use an otoscope for an examination. Which statement is true regarding the otoscope? The otoscope: a. Is often used to direct light onto the sinuses. b. Uses a short, broad speculum to help visualize the ear. c. Is used to examine the structures of the internal ear. d. Directs light into the ear canal and onto the tympanic membrane. D The otoscope directs light into the ear canal and onto the tympanic membrane that divides the external and middle ear. A short, broad speculum is used to visualize the nares. 18. An examiner is using an ophthalmoscope to examine a patient’s eyes. The patient has astigmatism and is nearsighted. The use of which of these techniques would indicate that the examination is being correctly performed? a. Using the large full circle of light when assessing pupils that are not dilated b. Rotating the lens selector dial to the black numbers to compensate for astigmatism c. Using the grid on the lens aperture dial to visualize the external structures of the eye d. Rotating the lens selector dial to bring the object into focus D The ophthalmoscope is used to examine the internal eye structures. It can compensate for nearsightedness or farsightedness, but it will not correct for astigmatism. The grid is used to assess size and location of lesions on the fundus. The large full spot of light is used to assess dilated pupils. Rotating the lens selector dial brings the object into focus. 19. The nurse is unable to palpate the right radial pulse on a patient. The best action would be to: a. Auscultate over the area with a fetoscope. b. Use a goniometer to measure the pulsations. c. Use a Doppler device to check for pulsations over the area. d. Check for the presence of pulsations with a stethoscope. C Doppler devices are used to augment pulse or blood pressure measurements. Goniometers measure joint range of motion. A fetoscope is used to auscultate fetal heart tones. Stethoscopes are used to auscultate breath, bowel, and heart sounds. 20. The nurse is preparing to perform a physical assessment. The correct action by the nurse is reflected by which statement? The nurse: a. Performs the examination from the left side of the bed. b. Examines tender or painful areas first to help relieve the patient’s anxiety. c. Follows the same examination sequence, regardless of the patient’s age or condition. d. Organizes the assessment to ensure that the patient does not change positions too often. D The steps of the assessment should be organized to ensure that the patient does not change positions too often. The sequence of the steps of the assessment may differ, depending on the age of the person and the examiner’s preference. Tender or painful areas should be assessed last. 21. A man is at the clinic for a physical examination. He states that he is “very anxious” about the physical examination. What steps can the nurse take to make him more comfortable? a. Appear unhurried and confident when examining him. b. Stay in the room when he undresses in case he needs assistance. c. Ask him to change into an examining gown and to take off his undergarments. d. Defer measuring vital signs until the end of the examination, which allows him time to become comfortable. A Anxiety can be reduced by an examiner who is confident, self-assured, considerate, and unhurried. Familiar and relatively nonthreatening actions, such as measuring the person’s vital signs, will gradually accustom the person to the examination. 22. When performing a physical examination, safety must be considered to protect the examiner and the patient against the spread of infection. Which of these statements describes the most appropriate action the nurse should take when performing a physical examination? a. Washing one’s hands after removing gloves is not necessary, as long as the gloves are still intact. b. Hands are washed before and after every physical patient encounter. c. Hands are washed before the examination of each body system to prevent the spread of bacteria from one part of the body to another. d. Gloves are worn throughout the entire examination to demonstrate to the patient concern regarding the spread of infectious diseases. B The nurse should wash his or her hands before and after every physical patient encounter; after contact with blood, body fluids, secretions, and excretions; after contact with any equipment contaminated with body fluids; and after removing gloves. Hands should be washed after gloves have been removed, even if the gloves appear to be intact. Gloves should be worn when potential contact with any body fluids is present. 23. The nurse is examining a patient’s lower leg and notices a draining ulceration. Which of these actions is most appropriate in this situation? a. Washing hands, and contacting the physician b. Continuing to examine the ulceration, and then washing hands c. Washing hands, putting on gloves, and continuing with the examination of the ulceration d. Washing hands, proceeding with rest of the physical examination, and then continuing with the examination of the leg ulceration C The examiner should wear gloves when the potential contact with any body fluids is present. In this situation, the nurse should wash his or her hands, put on gloves, and continue examining the ulceration. 24. During the examination, offering some brief teaching about the patient’s body or the examiner’s findings is often appropriate. Which one of these statements by the nurse is most appropriate? a. “Your atrial dysrhythmias are under control.” b. “You have pitting edema and mild varicosities.” c. “Your pulse is 80 beats per minute, which is within the normal range.” d. “I’m using my stethoscope to listen for any crackles, wheezes, or rubs.” C The sharing of some information builds rapport, as long as the patient is able to understand the terminology. 25. The nurse keeps in mind that the most important reason to share information and to offer brief teaching while performing the physical examination is to help the: a. Examiner feel more comfortable and to gain control of the situation. b. Examiner to build rapport and to increase the patient’s confidence in him or her. c. Patient understand his or her disease process and treatment modalities. d. Patient identify questions about his or her disease and the potential areas of patient education. B Sharing information builds rapport and increases the patient’s confidence in the examiner. It also gives the patient a little more control in a situation during which feeling completely helpless is often present. 26. The nurse is examining an infant and prepares to elicit the Moro reflex at which time during the examination? a. When the infant is sleeping b. At the end of the examination c. Before auscultation of the thorax d. Halfway through the examination B The Moro or startle reflex is elicited at the end of the examination because it may cause the infant to cry. 27. When preparing to perform a physical examination on an infant, the nurse should: a. Have the parent remove all clothing except the diaper on a boy. b. Instruct the parent to feed the infant immediately before the examination. c. Encourage the infant to suck on a pacifier during the abdominal examination. d. Ask the parent to leave the room briefly when assessing the infant’s vital signs. A The parent should always be present to increase the child’s feeling of security and to understand normal growth and development. The timing of the examination should be 1 to 2 hours after feeding when the baby is neither too drowsy nor too hungry. Infants do not object to being nude; clothing should be removed, but a diaper should be left on a boy. 28. A 6-month-old infant has been brought to the well-child clinic for a check-up. She is currently sleeping. What should the nurse do first when beginning the examination? a. Auscultate the lungs and heart while the infant is still sleeping. b. Examine the infant’s hips, because this procedure is uncomfortable. c. Begin with the assessment of the eye, and continue with the remainder of the examination in a head-to-toe approach. d. Wake the infant before beginning any portion of the examination to obtain the most accurate assessment of body systems. A When the infant is quiet or sleeping is an ideal time to assess the cardiac, respiratory, and abdominal systems. Assessment of the eye, ear, nose, and throat are invasive procedures that should be performed at the end of the examination. 29. A 2-year-old child has been brought to the clinic for a well-child checkup. The best way for the nurse to begin the assessment is to: a. Ask the parent to place the child on the examining table. b. Have the parent remove all of the child’s clothing before the examination. c. Allow the child to keep a security object such as a toy or blanket during the examination. d. Initially focus the interactions on the child, essentially ignoring the parent until the child’s trust has been obtained. C The best place to examine the toddler is on the parent’s lap. Toddlers understand symbols; therefore, a security object is helpful. Initially, the focus is more on the parent, which allows the child to adjust gradually and to become familiar with you. A 2-year-old child does not like to take off his or her clothes. Therefore, ask the parent to undress one body part at a time. 30. The nurse is examining a 2-year-old child and asks, “May I listen to your heart now?” Which critique of the nurse’s technique is most accurate? a. Asking questions enhances the child’s autonomy b. Asking the child for permission helps develop a sense of trust c. This question is an appropriate statement because children at this age like to have choices d. Children at this age like to say, “No.” The examiner should not offer a choice when no choice is available D Children at this age like to say, “No.” Choices should not be offered when no choice is really available. If the child says, “No” and the nurse does it anyway, then the nurse loses trust. Autonomy is enhanced by offering a limited option, “Shall I listen to your heart next or your tummy?” 31. With which of these patients would it be most appropriate for the nurse to use games during the assessment, such as having the patient “blow out” the light on the penlight? a. Infant b. Preschool child c. School-age child d. Adolescent B When assessing preschool children, using games or allowing them to play with the equipment to reduce their fears can be helpful. Such games are not appropriate for the other age groups. 32. The nurse is preparing to examine a 4-year-old child. Which action is appropriate for this age group? a. Explain the procedures in detail to alleviate the child’s anxiety. b. Give the child feedback and reassurance during the examination. c. Do not ask the child to remove his or her clothes because children at this age are usually very private. d. Perform an examination of the ear, nose, and throat first, and then examine the thorax and abdomen. B With preschool children, short, simple explanations should be used. Children at this age are usually willing to undress. An examination of the head should be performed last. During the examination, needed feedback and reassurance should be given to the preschooler. 33. When examining a 16-year-old male teenager, the nurse should: a. Discuss health teaching with the parent because the teen is unlikely to be interested in promoting wellness. b. Ask his parent to stay in the room during the history and physical examination to answer any questions and to alleviate his anxiety. c. Talk to him the same manner as one would talk to a younger child because a teen’s level of understanding may not match his or her speech. d. Provide feedback that his body is developing normally, and discuss the wide variation among teenagers on the rate of growth and development. D During the examination, the adolescent needs feedback that his or her body is healthy and developing normally. The adolescent has a keen awareness of body image and often compares him or herself with peers. Apprise the adolescent of the wide variation among teenagers on the rate of growth and development. 34. When examining an older adult, the nurse should use which technique? a. Avoid touching the patient too much. b. Attempt to perform the entire physical examination during one visit. c. Speak loudly and slowly because most aging adults have hearing deficits. d. Arrange the sequence of the examination to allow as few position changes as possible. D When examining the older adult, arranging the sequence of the examination to allow as few position changes as possible is best. Physical touch is especially important with the older person because other senses may be diminished. 35. The most important step that the nurse can take to prevent the transmission of microorganisms in the hospital setting is to: a. Wear protective eye wear at all times. b. Wear gloves during any and all contact with patients. c. Wash hands before and after contact with each patient. d. Clean the stethoscope with an alcohol swab between patients. C The most important step to decrease the risk of microorganism transmission is to wash hands promptly and thoroughly before and after physical contact with each patient. Stethoscopes should also be cleansed with an alcohol swab before and after each patient contact. The best routine is to combine stethoscope rubbing with hand hygiene each time hand hygiene is performed. 36. Which of these statements is true regarding the use of Standard Precautions in the health care setting? a. Standard Precautions apply to all body fluids, including sweat. b. Use alcohol-based hand rub if hands are visibly dirty. c. Standard Precautions are intended for use with all patients, regardless of their risk or presumed infection status. d. Standard Precautions are to be used only when nonintact skin, excretions containing visible blood, or expected contact with mucous membranes is present. C Standard Precautions are designed to reduce the risk of transmission of microorganisms from both recognized and unrecognized sources and are intended for use for all patients, regardless of their risk or presumed infection status. Standard Precautions apply to blood and all other body fluids, secretions and excretions except sweat—regardless of whether they contain visible blood, nonintact skin, or mucous membranes. Hands should be washed with soap and water if visibly soiled with blood or body fluids. Alcohol-based hand rubs can be used if hands are not visibly soiled. 37. The nurse is preparing to assess a hospitalized patient who is experiencing significant shortness of breath. How should the nurse proceed with the assessment? a. The patient should lie down to obtain an accurate cardiac, respiratory, and abdominal assessment. b. A thorough history and physical assessment information should be obtained from the patient’s family member. c. A complete history and physical assessment should be immediately performed to obtain baseline information. d. Body areas appropriate to the problem should be examined and then the assessment completed after the problem has resolved. D Both altering the position of the patient during the examination and collecting a mini database by examining the body areas appropriate to the problem may be necessary in this situation. An assessment may be completed later after the distress is resolved. 38. When examining an infant, the nurse should examine which area first? a. Ear b. Nose c. Throat d. Abdomen D The least-distressing steps are performed first, saving the invasive steps of the examination of the eye, ear, nose, and throat until last. 39. While auscultating heart sounds, the nurse hears a murmur. Which of these instruments should be used to assess this murmur? a. Electrocardiogram b. Bell of the stethoscope c. Diaphragm of the stethoscope d. Palpation with the nurse’s palm of the hand B The bell of the stethoscope is best for soft, low-pitched sounds such as extra heart sounds or murmurs. The diaphragm of the stethoscope is best used for high-pitched sounds such as breath, bowel, and normal heart sounds. 40. During an examination of a patient’s abdomen, the nurse notes that the abdomen is rounded and firm to the touch. During percussion, the nurse notes a drumlike quality of the sounds across the quadrants. This type of sound indicates: a. Constipation. b. Air-filled areas. c. Presence of a tumor. d. Presence of dense organs. B A musical or drumlike sound (tympany) is heard when percussion occurs over an air-filled viscus, such as the stomach or intestines. 41. The nurse is preparing to examine a 6-year-old child. Which action is most appropriate? a. The thorax, abdomen, and genitalia are examined before the head. b. Talking about the equipment being used is avoided because doing so may increase the child’s anxiety. c. The nurse should keep in mind that a child at this age will have a sense of modesty. d. The child is asked to undress from the waist up. C A 6-year-old child has a sense of modesty. The child should undress him or herself, leaving underpants on and using a gown or drape. A school-age child is curious to know how equipment works, and the sequence should progress from the child’s head to the toes. 42. During auscultation of a patient’s heart sounds, the nurse hears an unfamiliar sound. The nurse should: a. Document the findings in the patient’s record. b. Wait 10 minutes, and auscultate the sound again. c. Ask the patient how he or she is feeling. d. Ask another nurse to double check the finding. D If an abnormal finding is not familiar, then the nurse may ask another examiner to double check the finding. The other responses do not help identify the unfamiliar sound. MULTIPLE RESPONSE 1. The nurse is preparing to palpate the thorax and abdomen of a patient. Which of these statements describes the correct technique for this procedure? Select all that apply. a. Warm the hands first before touching the patient. b. For deep palpation, use one long continuous palpation when assessing the liver. c. Start with light palpation to detect surface characteristics. d. Use the fingertips to examine skin texture, swelling, pulsation, and presence of lumps. e. Identify any tender areas, and palpate them last. f. Use the palms of the hands to assess temperature of the skin. A, C, D, E The hands should always be warmed before beginning palpation. Intermittent pressure rather than one long continuous palpation is used; any tender areas are identified and palpated last. Fingertips are used to examine skin texture, swelling, pulsation, and the presence of lumps. The dorsa (backs) of the hands are used to assess skin temperature because the skin on the dorsa is thinner than on the palms. CHAPTER 9 1. The nurse is performing a general survey. Which action is a component of the general survey? a. Observing the patient’s body stature and nutritional status b. Interpreting the subjective information the patient has reported c. Measuring the patient’s temperature, pulse, respirations, and blood pressure d. Observing specific body systems while performing the physical assessment A The general survey is a study of the whole person that includes observing the patient’s physical appearance, body structure, mobility, and behavior. 2. When measuring a patient’s weight, the nurse is aware of which of these guidelines? a. The patient is always weighed wearing only his or her undergarments. b. The type of scale does not matter, as long as the weights are similar from day to day. c. The patient may leave on his or her jacket and shoes as long as these are documented next to the weight. d. Attempts should be made to weigh the patient at approximately the same time of day, if a sequence of weights is necessary. D A standardized balance scale is used to measure weight. The patient should remove his or her shoes and heavy outer clothing. If a sequence of repeated weights is necessary, then the nurse should attempt to weigh the patient at approximately the same time of day and with the same types of clothing worn each time. 3. A patient’s weekly blood pressure readings for 2 months have ranged between 124/84 mm Hg and 136/88 mm Hg, with an average reading of 126/86 mm Hg. The nurse knows that this blood pressure falls within which blood pressure category? a. Normal blood pressure b. Prehypertension c. Stage 1 hypertension d. Stage 2 hypertension B According to the Seventh Report of the Joint National Committee (JNC 7) guidelines, prehypertension blood pressure readings are systolic readings of 120 to 139 mm Hg or diastolic readings of 50 to 89 mm Hg. 4. During an examination of a child, the nurse considers that physical growth is the best index of a child’s: a. General health. b. Genetic makeup. c. Nutritional status. d. Activity and exercise patterns. A Physical growth is the best index of a child’s general health; recording the child’s height and weight helps determine normal growth patterns. 5. A 1-month-old infant has a head measurement of 34 cm and has a chest circumference of 32 cm. Based on the interpretation of these findings, the nurse would: a. Refer the infant to a physician for further evaluation. b. Consider these findings normal for a 1-month-old infant. c. Expect the chest circumference to be greater than the head circumference. d. Ask the parent to return in 2 weeks to re-evaluate the head and chest circumferences. B The newborn’s head measures approximately 32 to 38 cm and is approximately 2 cm larger than the chest circumference. Between 6 months and 2 years, both measurements are approximately the same, and after age 2 years, the chest circumference is greater than the head circumference. 6. The nurse is assessing an 80-year-old male patient. Which assessment findings would be considered normal? a. Increase in body weight from his younger years b. Additional deposits of fat on the thighs and lower legs c. Presence of kyphosis and flexion in the knees and hips d. Change in overall body proportion, including a longer trunk and shorter extremities C Changes that occur in the aging person include more prominent bony landmarks, decreased body weight (especially in men), a decrease in subcutaneous fat from the face and periphery, and additional fat deposited on the abdomen and hips. Postural changes of kyphosis and slight flexion in the knees and hips also occur. 7. The nurse should measure rectal temperatures in which of these patients? a. School-age child b. Older adult c. Comatose adult d. Patient receiving oxygen by nasal cannula C Rectal temperatures should be taken when the other routes are impractical, such as for comatose or confused persons, for those in shock, or for those who cannot close the mouth because of breathing or oxygen tubes, a wired mandible, or other facial dysfunctions. 8. The nurse is preparing to measure the length, weight, chest, and head circumference of a 6-month-old infant. Which measurement technique is correct? a. Measuring the infant’s length by using a tape measure b. Weighing the infant by placing him or her on an electronic standing scale c. Measuring the chest circumference at the nipple line with a tape measure d. Measuring the head circumference by wrapping the tape measure over the nose and cheekbones C To measure the chest circumference, the tape is encircled around the chest at the nipple line. The length should be measured on a horizontal measuring board. Weight should be measured on a platform-type balance scale. Head circumference is measured with the tape around the head, aligned at the eyebrows, and at the prominent frontal and occipital bones—the widest span is correct. 9. The nurse knows that one advantage of the tympanic membrane thermometer (TMT) is that: a. Rapid measurement is useful for uncooperative younger children. b. Using the TMT is the most accurate method for measuring body temperature in newborn infants. c. Measuring temperature using the TMT is inexpensive. d. Studies strongly support the use of the TMT in children under the age 6 years. A The TMT is useful for young children who may not cooperate for oral temperatures and fear rectal temperatures. However, the use a TMT with newborn infants and young children is conflicting. 10. When assessing an older adult, which vital sign changes occur with aging? a. Increase in pulse rate b. Widened pulse pressure c. Increase in body temperature d. Decrease in diastolic blood pressure B With aging, the nurse keeps in mind that the systolic blood pressure increases, leading to widened pulse pressure. With many older people, both the systolic and diastolic pressures increase. The pulse rate and temperature do not increase. 11. The nurse is examining a patient who is complaining of “feeling cold.” Which is a mechanism of heat loss in the body? a. Exercise b. Radiation c. Metabolism d. Food digestion B The body maintains a steady temperature through a thermostat or feedback mechanism, which is regulated in the hypothalamus of the brain. The hypothalamus regulates heat production from metabolism, exercise, food digestion, and external factors with heat loss through radiation, evaporation of sweat, convection, and conduction. 12. When measuring a patient’s body temperature, the nurse keeps in mind that body temperature is influenced by: a. Constipation. b. Patient’s emotional state. c. Diurnal cycle. d. Nocturnal cycle. C Normal temperature is influenced by the diurnal cycle, exercise, and age. The other responses do not influence body temperature. 13. When evaluating the temperature of older adults, the nurse should remember which aspect about an older adult’s body temperature? a. The body temperature of the older adult is lower than that of a younger adult. b. An older adult’s body temperature is approximately the same as that of a young child. c. Body temperature depends on the type of thermometer used. d. In the older adult, the body temperature varies widely because of less effective heat control mechanisms. A In older adults, the body temperature is usually lower than in other age groups, with a mean temperature of 36.2° C. 14. A 60-year-old male patient has been treated for pneumonia for the past 6 weeks. He is seen today in the clinic for an “unexplained” weight loss of 10 pounds over the last 6 weeks. The nurse knows that: a. Weight loss is probably the result of unhealthy eating habits. b. Chronic diseases such as hypertension cause weight loss. c. Unexplained weight loss often accompanies short-term illnesses. d. Weight loss is probably the result of a mental health dysfunction. C An unexplained weight loss may be a sign of a short-term illness or a chronic illness such as endocrine disease, malignancy, depression, anorexia nervosa, or bulimia. 15. When assessing a 75-year-old patient who has asthma, the nurse notes that he assumes a tripod position, leaning forward with arms braced on the chair. On the basis of this observation, the nurse should: a. Assume that the patient is eager and interested in participating in the interview. b. Evaluate the patient for abdominal pain, which may be exacerbated in the sitting position. c. Assume that the patient is having difficulty breathing and assist him to a supine position. d. Recognize that a tripod position is often used when a patient is having respiratory difficulties. D Assuming a tripod position—leaning forward with arms braced on chair arms—occurs with chronic pulmonary disease. The other actions or assumptions are not correct. 16. Which of these actions illustrates the correct technique the nurse should use when assessing oral temperature with a mercury thermometer? a. Wait 30 minutes if the patient has ingested hot or iced liquids. b. Leave the thermometer in place 3 to 4 minutes if the patient is afebrile. c. Place the thermometer in front of the tongue, and ask the patient to close his or her lips. d. Shake the mercury-in-glass thermometer down to below 36.6° C before taking the temperature. B The thermometer should be left in place 3 to 4 minutes if the person is afebrile and up to 8 minutes if the person is febrile. The nurse should wait 15 minutes if the person has just ingested hot or iced liquids and 2 minutes if he or she has just smoked. 17. The nurse is taking temperatures in a clinic with a TMT. Which statement is true regarding use of the TMT? a. A tympanic temperature is more time consuming than a rectal temperature. b. The tympanic method is more invasive and uncomfortable than the oral method. c. The risk of cross-contamination is reduced, compared with the rectal route. d. The tympanic membrane most accurately reflects the temperature in the ophthalmic artery. C The TMT is a noninvasive, nontraumatic device that is extremely quick and efficient. The chance of cross-contamination with the TMT is minimal because the ear canal is lined with skin, not mucous membranes. 18. To assess a rectal temperature accurately in an adult, the nurse would: a. Use a lubricated blunt tip thermometer. b. Insert the thermometer 2 to 3 inches into the rectum. c. Leave the thermometer in place up to 8 minutes if the patient is febrile. d. Wait 2 to 3 minutes if the patient has recently smoked a cigarette. A A lubricated rectal thermometer (with a short, blunt tip) is inserted only 2 to 3 cm (1 inch) into the adult rectum and left in place for 2 minutes. Cigarette smoking does not alter rectal temperatures. 19. Which technique is correct when the nurse is assessing the radial pulse of a patient? The pulse is counted for: a. 1 minute, if the rhythm is irregular. b. 15 seconds and then multiplied by 4, if the rhythm is regular. c. 2 full minutes to detect any variation in amplitude. d. 10 seconds and then multiplied by 6, if the patient has no history of cardiac abnormalities. A Recent research suggests that the 30-second interval multiplied by 2 is the most accurate and efficient technique when heart rates are normal or rapid and when rhythms are regular. If the rhythm is irregular, then the pulse is counted for 1 full minute. 20. When assessing a patient’s pulse, the nurse should also notice which of these characteristics? a. Force b. Pallor c. Capillary refill time d. Timing in the cardiac cycle A The pulse is assessed for rate, rhythm, and force. 21. When assessing the pulse of a 6-year-old boy, the nurse notices that his heart rate varies with his respiratory cycle, speeding up at the peak of inspiration and slowing to normal with expiration. The nurse’s next action would be to: a. Immediately notify the physician. b. Consider this finding normal in children and young adults. c. Check the child’s blood pressure, and note any variation with respiration. d. Document that this child has bradycardia, and continue with the assessment. B Sinus arrhythmia is commonly found in children and young adults. During the respiratory cycle, the heart rate varies, speeding up at the peak of inspiration and slowing to normal with expiration. 22. When assessing the force, or strength, of a pulse, the nurse recalls that the pulse: a. Is usually recorded on a 0- to 2-point scale. b. Demonstrates elasticity of the vessel wall. c. Is a reflection of the heart’s stroke volume. d. Reflects the blood volume in the arteries during diastole. C The heart pumps an amount of blood (the stroke volume) into the aorta. The force flares the arterial walls and generates a pressure wave, which is felt in the periphery as the pulse. 23. The nurse is assessing the vital signs of a 20-year-old male marathon runner and documents the following vital signs: temperature–36° C; pulse–48 beats per minute; respirations–14 breaths per minute; blood pressure–104/68 mm Hg. Which statement is true concerning these results? a. The patient is experiencing tachycardia. b. These are normal vital signs for a healthy, athletic adult. c. The patient’s pulse rate is not normal—his physician should be notified. d. On the basis of these readings, the patient should return to the clinic in 1 week. B In the adult, a heart rate less than 50 beats per minute is called bradycardia, which normally occurs in the well-trained athlete whose heart muscle develops along with the skeletal muscles. 24. The nurse is assessing the vital signs of a 3-year-old patient who appears to have an irregular respiratory pattern. How should the nurse assess this child’s respirations? a. Respirations should be counted for 1 full minute, noticing rate and rhythm. b. Child’s pulse and respirations should be simultaneously checked for 30 seconds. c. Child’s respirations should be checked for a minimum of 5 minutes to identify any variations in his or her respiratory pattern. d. Patient’s respirations should be counted for 15 seconds and then multiplied by 4 to obtain the number of respirations per minute. A Respirations are counted for 1 full minute if an abnormality is suspected. The other responses are not correct actions. 25. A patient’s blood pressure is 118/82 mm Hg. He asks the nurse, “What do the numbers mean?” The nurse’s best reply is: a. “The numbers are within the normal range and are nothing to worry about.” b. “The bottom number is the diastolic pressure and reflects the stroke volume of the heart.” c. “The top number is the systolic blood pressure and reflects the pressure of the blood against the arteries when the heart contracts.” d. “The concept of blood pressure is difficult to understand. The primary thing to be concerned about is the top number, or the systolic blood pressure.” C The systolic pressure is the maximum pressure felt on the artery during left ventricular contraction, or systole. The diastolic pressure is the elastic recoil, or resting, pressure that the blood constantly exerts in between each contraction. The nurse should answer the patient’s question and use terms he can understand. 26. While measuring a patient’s blood pressure, the nurse recalls that certain factors, such as __________, help determine blood pressure. a. Pulse rate b. Pulse pressure c. Vascular output d. Peripheral vascular resistance D The level of blood pressure is determined by five factors: cardiac output, peripheral vascular resistance, volume of circulating blood, viscosity, and elasticity of the vessel walls. 27. A nurse is helping at a health fair at a local mall. When taking blood pressures on a variety of people, the nurse keeps in mind that: a. After menopause, blood pressure readings in women are usually lower than those taken in men. b. The blood pressure of a Black adult is usually higher than that of a White adult of the same age. c. Blood pressure measurements in people who are overweight should be the same as those of people who are at a normal weight. d. A teenager’s blood pressure reading will be lower than that of an adult. B In the United States, a Black adult’s blood pressure is usually higher than that of a White adult of the same age. The incidence of hypertension is twice as high in Blacks as it is in Whites. After menopause, blood pressure in women is higher than in men; blood pressure measurements in people who are obese are usually higher than in those who are not overweight. Normally, a gradual rise occurs through childhood and into the adult years. 28. The nurse notices a colleague is preparing to check the blood pressure of a patient who is obese by using a standard-sized blood pressure cuff. The nurse should expect the reading to: a. Yield a falsely low blood pressure. b. Yield a falsely high blood pressure. c. Be the same, regardless of cuff size. d. Vary as a result of the technique of the person performing the assessment. B Using a cuff that is too narrow yields a falsely high blood pressure because it takes extra pressure to compress the artery. 29. A student is late for his appointment and has rushed across campus to the health clinic. The nurse should: a. Allow 5 minutes for him to relax and rest before checking his vital signs. b. Check the blood pressure in both arms, expecting a difference in the readings because of his recent exercise. c. Immediately monitor his vital signs on his arrival at the clinic and then 5 minutes later, recording any differences. d. Check his blood pressure in the supine position, which will provide a more accurate reading and will allow him to relax at the same time. A A comfortable, relaxed person yields a valid blood pressure. Many people are anxious at the beginning of an examination; the nurse should allow at least a 5-minute rest period before measuring blood pressure. 30. The nurse will perform a palpated pressure before auscultating blood pressure. The reason for this is to: a. More clearly hear the Korotkoff sounds. b. Detect the presence of an auscultatory gap. c. Avoid missing a falsely elevated blood pressure. d. More readily identify phase IV of the Korotkoff sounds. : B Inflation of the cuff 20 to 30 mm Hg beyond the point at which a palpated pulse disappears will avoid missing an auscultatory gap, which is a period when the Korotkoff sounds disappear during auscultation. 31. The nurse is taking an initial blood pressure reading on a 72-year-old patient with documented hypertension. How should the nurse proceed? a. Cuff should be placed on the patient’s arm and inflated 30 mm Hg above the patient’s pulse rate. b. Cuff should be inflated to 200 mm Hg in an attempt to obtain the most accurate systolic reading. c. Cuff should be inflated 30 mm Hg above the point at which the palpated pulse disappears. d. After confirming the patient’s previous blood pressure readings, the cuff should be inflated 30 mm Hg above the highest systolic reading recorded. C An auscultatory gap occurs in approximately 5% of the people, most often in those with hypertension. To check for the presence of an auscultatory gap, the cuff should be inflated 20 to 30 mm Hg beyond the point at which the palpated pulse disappears. 32. The nurse has collected the following information on a patient: palpated blood pressure–180 mm Hg; auscultated blood pressure–170/100 mm Hg; apical pulse–60 beats per minute; radial pulse–70 beats per minute. What is the patient’s pulse pressure? a. 10 b. 70 c. 80 d. 100 B Pulse pressure is the difference between systolic and diastolic blood pressure (170 – 100 = 70) and reflects the stroke volume. 33. When auscultating the blood pressure of a 25-year-old patient, the nurse notices the phase I Korotkoff sounds begin at 200 mm Hg. At 100 mm Hg, the Korotkoff sounds muffle. At 92 mm Hg, the Korotkoff sounds disappear. How should the nurse record this patient’s blood pressure? a. 200/92 b. 200/100 c. 100/200/92 d. 200/100/92 A In adults, the last audible sound best indicates the diastolic pressure. When a variance is greater than 10 to 12 mm Hg between phases IV and V, both phases should be recorded along with the systolic reading (e.g., 142/98/80). 34. A patient is seen in the clinic for complaints of “fainting episodes that started last week.” How should the nurse proceed with the examination? a. Blood pressure readings are taken in both the arms and the thighs. b. The patient is assisted to a lying position, and his blood pressure is taken. c. His blood pressure is recorded in the lying, sitting, and standing positions. d. His blood pressure is recorded in the lying and sitting positions; these numbers are then averaged to obtain a mean blood pressure. C If the person is known to have hypertension, is taking antihypertensive medications, or reports a history of fainting or syncope, then the blood pressure reading should be taken in three positions: lying, sitting, and standing. 35. A 70-year-old man has a blood pressure of 150/90 mm Hg in a lying position, 130/80 mm Hg in a sitting position, and 100/60 mm Hg in a standing position. How should the nurse evaluate these findings? a. These readings are a normal response and attributable to changes in the patient’s position. b. The change in blood pressure readings is called orthostatic hypotension. c. The blood pressure reading in the lying position is within normal limits. d. The change in blood pressure readings is considered within normal limits for the patient’s age. B Orthostatic hypotension is a drop in systolic pressure of more than 20 mm Hg, which occurs with a quick change to a standing position. Aging people have the greatest risk of this problem. 36. The nurse is helping another nurse to take a blood pressure reading on a patient’s thigh. Which action is correct regarding thigh pressure? a. Either the popliteal or femoral vessels should be auscultated to obtain a thigh pressure. b. The best position to measure thigh pressure is the supine position with the knee slightly bent. c. If the blood pressure in the arm is high in an adolescent, then it should be compared with the thigh pressure. d. The thigh pressure is lower than the pressure in the arm, which is attributable to the distance away from the heart and the size of the popliteal vessels. C When blood pressure measured at the arm is excessively high, particularly in adolescents and young adults, it is compared with thigh pressure to check for coarctation of the aorta. The popliteal artery is auscultated for the reading. Generally, thigh pressure is higher than that of the arm; however, if coarctation of the artery is present, then arm pressures are higher than thigh pressures. 37. The nurse is preparing to measure the vital signs of a 6-month-old infant. Which action by the nurse is correct? a. Respirations are measured; then pulse and temperature. b. Vital signs should be measured more frequently than in an adult. c. Procedures are explained to the parent, and the infant is encouraged to handle the equipment. d. The nurse should first perform the physical examination to allow the infant to become more familiar with her and then measure the infant’s vital signs. A With an infant, the order of vital sign measurements is reversed to respiration, pulse, and temperature. Taking the temperature first, especially if it is rectal, may cause the infant to cry, which will increase the respiratory and pulse rate, thus masking the normal resting values. The vital signs are measured with the same purpose and frequency as would be measured in an adult. 38. A 4-month-old child is at the clinic for a well-baby check-up and immunizations. Which of these actions is most appropriate when the nurse is assessing an infant’s vital signs? a. The infant’s radial pulse should be palpated, and the nurse should notice any fluctuations resulting from activity or exercise. b. The nurse should auscultate an apical rate for 1 minute and then assess for any normal irregularities, such as sinus arrhythmia. c. The infant’s blood pressure should be assessed by using a stethoscope with a large diaphragm piece to hear the soft muffled Korotkoff sounds. d. The infant’s chest should be observed and the respiratory rate counted for 1 minute; the respiratory pattern may vary significantly. B The nurse palpates or auscultates an apical rate with infants and toddlers. The pulse should be counted for 1 full minute to account for normal irregularities, such as sinus arrhythmia. Children younger than 3 years of age have such small arm vessels; consequently, hearing Korotkoff sounds with a stethoscope is difficult. The nurse should use either an electronic blood pressure device that uses oscillometry or a Doppler ultrasound device to amplify the sounds. 39. The nurse is conducting a health fair for older adults. Which statement is true regarding vital sign measurements in aging adults? a. The pulse is more difficult to palpate because of the stiffness of the blood vessels. b. An increased respiratory rate and a shallower inspiratory phase are expected findings. c. A decreased pulse pressure occurs from changes in the systolic and diastolic blood pressures. d. Changes in the body’s temperature regulatory mechanism leave the older person more likely to develop a fever. B Aging causes a decrease in vital capacity and decreased inspiratory reserve volume. The examiner may notice a shallower inspiratory phase and an increased respiratory rate. An increase in the rigidity of the arterial walls makes the pulse actually easier to palpate. Pulse pressure is widened in older adults, and changes in the body temperature regulatory mechanism leave the older person less likely to have fever but at a greater risk for hypothermia. 40. In a patient with acromegaly, the nurse will expect to discover which assessment findings? a. Heavy, flattened facial features b. Growth retardation and a delayed onset of puberty c. Overgrowth of bone in the face, head, hands, and feet d. Increased height and weight and delayed sexual development C Excessive secretions of growth hormone in adulthood after normal completion of body growth causes an overgrowth of the bones in the face, head, hands, and feet but no change in height. 41. The nurse is performing a general survey of a patient. Which finding is considered normal? a. When standing, the patient’s base is narrow. b. The patient appears older than his stated age. c. Arm span (fingertip to fingertip) is greater than the height. d. Arm span (fingertip to fingertip) equals the patient’s height. D When performing the general survey, the patient’s arm span (fingertip to fingertip) should equal the patient’s height. An arm span that is greater than the person’s height may indicate Marfan syndrome. The base should be wide when the patient is standing, and an older appearance than the stated age may indicate a history of a chronic illness or chronic alcoholism. 42. The nurse is assessing children in a pediatric clinic. Which statement is true regarding the measurement of blood pressure in children? a. Blood pressure guidelines for children are based on age. b. Phase II Korotkoff sounds are the best indicator of systolic blood pressure in children. c. Using a Doppler device is recommended for accurate blood pressure measurements until adolescence. d. The disappearance of phase V Korotkoff sounds can be used for the diastolic reading in children. D The disappearance of phase V Korotkoff sounds can be used for the diastolic reading in children, as well as in adults. 43. What type of blood pressure measurement error is most likely to occur if the nurse does not check for the presence of an auscultatory gap? a. Diastolic blood pressure may not be heard. b. Diastolic blood pressure may be falsely low. c. Systolic blood pressure may be falsely low. d. Systolic blood pressure may be falsely high. C If an auscultatory gap is undetected, then a falsely low systolic or falsely high diastolic reading may result, which is common in patients with hypertension. 44. When considering the concepts related to blood pressure, the nurse knows that the concept of mean arterial pressure (MAP) is best described by which statement? a. MAP is the pressure of the arterial pulse. b. MAP reflects the stroke volume of the heart. c. MAP is the pressure forcing blood into the tissues, averaged over the cardiac cycle. d. MAP is an average of the systolic and diastolic blood pressures and reflects tissue perfusion. C MAP is the pressure that forces blood into the tissues, averaged over the cardiac cycle. Stroke volume is reflected by the blood pressure. MAP is not an arithmetic average of systolic and diastolic pressures because diastole lasts longer; rather, it is a value closer to diastolic pressure plus one third of the pulse pressure. 45. A 75-year-old man with a history of hypertension was recently changed to a new antihypertensive drug. He reports feeling dizzy at times. How should the nurse evaluate his blood pressure? a. Blood pressure and pulse should be recorded in the supine, sitting, and standing positions. b. The patient should be directed to walk around the room and his blood pressure assessed after this activity. c. Blood pressure and pulse are assessed at the beginning and at the end of the examination. d. Blood pressure is taken on the right arm and then 5 minutes later on the left arm. A Orthostatic vital signs should be taken when the person is hypertensive or is taking antihypertensive medications, when the person reports fainting or syncope, or when volume depletion is suspected. The blood pressure and pulse readings are recorded in the supine, sitting, and standing positions. 46. Which of these specific measurements is the best index of a child’s general health? a. Vital signs b. Height and weight c. Head circumference d. Chest circumference B Physical growth, measured by height and weight, is the best index of a child’s general health. 47. The nurse is assessing an 8-year-old child whose growth rate measures below the third percentile for a child his age. He appears significantly younger than his stated age and is chubby with infantile facial features. Which condition does this child have? a. Hypopituitary dwarfism b. Achondroplastic dwarfism c. Marfan syndrome d. Acromegaly A Hypopituitary dwarfism is caused by a deficiency in growth hormone in childhood and results in a retardation of growth below the third percentile, delayed puberty, and other problems. The child’s appearance fits this description. Achondroplastic dwarfism is a genetic disorder resulting in characteristic deformities; Marfan syndrome is an inherited connective tissue disorder characterized by a tall, thin stature and other features. Acromegaly is the result of excessive secretion of growth hormone in adulthood. (For more information, see Table 9-5, Abnormalities in Body Height and Proportion.) 48. The nurse is counting an infant’s respirations. Which technique is correct? a. Watching the chest rise and fall b. Watching the abdomen for movement c. Placing a hand across the infant’s chest d. Using a stethoscope to listen to the breath sounds B Watching the abdomen for movement is the correct technique because the infant’s respirations are normally more diaphragmatic than thoracic. The other responses do not reflect correct techniques. 49. When checking for proper blood pressure cuff size, which guideline is correct? a. The standard cuff size is appropriate for all sizes. b. The length of the rubber bladder should equal 80% of the arm circumference. c. The width of the rubber bladder should equal 80% of the arm circumference. d. The width of the rubber bladder should equal 40% of the arm circumference. D The width of the rubber bladder should equal 40% of the circumference of the person's arm. The length of the bladder should equal 80% of this circumference. 50. During an examination, the nurse notices that a female patient has a round “moon” face, central trunk obesity, and a cervical hump. Her skin is fragile with bruises. The nurse determines that the patient has which condition? a. Marfan syndrome b. Gigantism c. Cushing syndrome d. Acromegaly C Cushing syndrome is characterized by weight gain and edema with central trunk and cervical obesity (buffalo hump) and round plethoric face (moon face). Excessive catabolism causes muscle wasting; weakness; thin arms and legs; reduced height; and thin, fragile skin with purple abdominal striae, bruising, and acne. (See Table 9-5, Abnormalities in Body Height and Proportion, for the definitions of the other conditions.) MULTIPLE RESPONSE 1. While measuring a patient’s blood pressure, the nurse uses the proper technique to obtain an accurate reading. Which of these situations will result in a falsely high blood pressure reading? Select all that apply. a. The person supports his or her own arm during the blood pressure reading. b. The blood pressure cuff is too narrow for the extremity. c. The arm is held above level of the heart. d. The cuff is loosely wrapped around the arm. e. The person is sitting with his or her legs crossed. f. The nurse does not inflate the cuff high enough. A, B, D, E Several factors can result in blood pressure readings that are too high or too low. Having the patient’s arm held above the level of the heart is one part of the correct technique. (Refer to Table 9-5, Common Errors in Blood Pressure Measurement.) SHORT ANSWER 1. What is the pulse pressure for a patient whose blood pressure is 158/96 mm Hg and whose pulse rate is 72 beats per minute? 62 The pulse pressure is the difference between the systolic and diastolic and reflects the stroke volume. The pulse rate is not necessary for pulse pressure calculations. CHAPTER 11 1. The nurse recognizes which of these persons is at greatest risk for undernutrition? a. 5-month-old infant b. 50-year-old woman c. 20-year-old college student d. 30-year-old hospital administrator A Vulnerable groups for undernutrition are infants, children, pregnant women, recent immigrants, persons with low incomes, hospitalized people, and aging adults. 2. When assessing a patient’s nutritional status, the nurse recalls that the best definition of optimal nutritional status is sufficient nutrients that: a. Are in excess of daily body requirements. b. Provide for the minimum body needs. c. Provide for daily body requirements but do not support increased metabolic demands. d. Provide for daily body requirements and support increased metabolic demands. D Optimal nutritional status is achieved when sufficient nutrients are consumed to support day-to-day body needs and any increased metabolic demands resulting from growth, pregnancy, or illness. 3. The nurse is providing nutrition information to the mother of a 1-year-old child. Which of these statements represents accurate information for this age group? a. Maintaining adequate fat and caloric intake is important for a child in this age group. b. The recommended dietary allowances for an infant are the same as for an adolescent. c. The baby’s growth is minimal at this age; therefore, caloric requirements are decreased. d. The baby should be placed on skim milk to decrease the risk of coronary artery disease when he or she grows older. A Because of rapid growth, especially of the brain, both infants and children younger than 2 years of age should not drink skim or low-fat milk or be placed on low-fat diets. Fats (calories and essential fatty acids) are required for proper growth and central nervous system development. 4. A pregnant woman is interested in breastfeeding her baby and asks several questions about the topic. Which information is appropriate for the nurse to share with her? a. Breastfeeding is best when also supplemented with bottle feedings. b. Babies who are breastfed often require supplemental vitamins. c. Breastfeeding is recommended for infants for the first 2 years of life. d. Breast milk provides the nutrients necessary for growth, as well as natural immunity. D Breastfeeding is recommended for full-term infants for the first year of life because breast milk is ideally formulated to promote normal infant growth and development, as well as natural immunity. The other statements are not correct. 5. A mother and her 13-year-old daughter express their concern related to the daughter’s recent weight gain and her increase in appetite. Which of these statements represents information the nurse should discuss with them? a. Dieting and exercising are necessary at this age. b. Snacks should be high in protein, iron, and calcium. c. Teenagers who have a weight problem should not be allowed to snack. d. A low-calorie diet is important to prevent the accumulation of fat. B After a period of slow growth in late childhood, adolescence is characterized by rapid physical growth and endocrine and hormonal changes. Caloric and protein requirements increase to meet this demand. Because of bone growth and increasing muscle mass (and, in girls, the onset of menarche), calcium and iron requirements also increase. 6. The nurse is assessing a 30-year-old unemployed immigrant from an underdeveloped country who has been in the United States for 1 month. Which of these problems related to his nutritional status might the nurse expect to find? a. Obesity b. Hypotension c. Osteomalacia (softening of the bones) d. Coronary artery disease C General undernutrition, hypertension, diarrhea, lactose intolerance, osteomalacia, scurvy, and dental caries are among the more common nutrition-related problems of new immigrants from developing countries. 7. For the first time, the nurse is seeing a patient who has no history of nutrition-related problems. The initial nutritional screening should include which activity? a. Calorie count of nutrients b. Anthropometric measures c. Complete physical examination d. Measurement of weight and weight history D Parameters used for nutrition screening typically include weight and weight history, conditions associated with increased nutritional risk, diet information, and routine laboratory data. The other responses reflect a more in-depth assessment rather than a screening. 8. A patient is asked to indicate on a form how many times he eats a specific food. This method describes which of these tools for obtaining dietary information? a. Food diary b. Calorie count c. 24-hour recall d. Food-frequency questionnaire D With this tool, information is collected on how many times per day, week, or month the individual eats particular foods, which provides an estimate of usual intake. 9. The nurse is providing care for a 68-year-old woman who is complaining of constipation. What concern exists regarding her nutritional status? a. Absorption of nutrients may be impaired. b. Constipation may represent a food allergy. c. The patient may need emergency surgery to correct the problem. d. Gastrointestinal problems will increase her caloric demand. A Gastrointestinal symptoms such as vomiting, diarrhea, or constipation may interfere with nutrient intake or absorption. The other responses are not correct. 10. During a nutritional assessment, why is it important for the nurse to ask a patient what medications he or she is taking? a. Certain drugs can affect the metabolism of nutrients. b. The nurse needs to assess the patient for allergic reactions. c. Medications need to be documented in the record for the physician’s review. d. Medications can affect one’s memory and ability to identify food eaten in the last 24 hours. A Analgesics, antacids, anticonvulsants, antibiotics, diuretics, laxatives, antineoplastic drugs, steroids, and oral contraceptives are drugs that can interact with nutrients, impairing their digestion, absorption, metabolism, or use. The other responses are not correct. 11. A patient tells the nurse that his food simply does not have any taste anymore. The nurse’s best response would be: a. “That must be really frustrating.” b. “When did you first notice this change?” c. “My food doesn’t always have a lot of taste either.” d. “Sometimes that happens, but your taste will come back.” B With changes in appetite, taste, smell, or chewing or swallowing, the examiner should ask about the type of change and when the change occurred. These problems interfere with adequate nutrient intake. The other responses are not correct. 12. The nurse is performing a nutritional assessment on a 15-year-old girl who tells the nurse that she is “so fat.” Assessment reveals that she is 5 feet 4 inches and weighs 110 pounds. The nurse’s appropriate response would be: a. “How much do you think you should weigh?” b. “Don’t worry about it; you’re not that overweight.” c. “The best thing for you would be to go on a diet.” d. “I used to always think I was fat when I was your age.” A Adolescents’ increased body awareness and self-consciousness may cause eating disorders such as anorexia nervosa or bulimia, conditions in which the real or perceived body image does not favorably compare with an ideal image. The nurse should not belittle the adolescent’s feelings, provide unsolicited advice, or agree with her. 13. The nurse is discussing appropriate foods with the mother of a 3-year-old child. Which of these foods are recommended? a. Foods that the child will eat, no matter what they are b. Foods easy to hold such as hot dogs, nuts, and grapes c. Any foods, as long as the rest of the family is also eating them d. Finger foods and nutritious snacks that cannot cause choking D Small portions, finger foods, simple meals, and nutritious snacks help improve the dietary intake of young children. Foods likely to be aspirated should be avoided (e.g., hot dogs, nuts, grapes, round candies, popcorn). 14. The nurse is reviewing the nutritional assessment of an 82-year-old patient. Which of these factors will most likely affect the nutritional status of an older adult? a. Increase in taste and smell b. Living alone on a fixed income c. Change in cardiovascular status d. Increase in gastrointestinal motility and absorption B Socioeconomic conditions frequently affect the nutritional status of the aging adult; these factors should be closely evaluated. Physical limitations, income, and social isolation are frequent problems that interfere with the acquisition of a balanced diet. A decrease in taste and smell and decreased gastrointestinal motility and absorption occur with aging. Cardiovascular status is not a factor that affects an older adult’s nutritional status. 15. When considering a nutritional assessment, the nurse is aware that the most common anthropometric measurements include: a. Height and weight. b. Leg circumference. c. Skinfold thickness of the biceps. d. Hip and waist measurements. A The most commonly used anthropometric measures are height, weight, triceps skinfold thickness, elbow breadth, and arm and head circumferences. 16. If a 29-year-old woman weighs 156 pounds, and the nurse determines her ideal body weight to be 120 pounds, then how would the nurse classify the woman’s weight? a. Obese b. Mildly overweight c. Suffering from malnutrition d. Within appropriate range of ideal weight A Obesity, as a result of caloric excess, refers to weight more than 20% above ideal body weight. For this patient, 20% of her ideal body weight would be 24 pounds, and greater than 20% of her body weight would be over 144 pounds. Therefore, having a weight of 156 pounds would be considered obese. 17. How should the nurse perform a triceps skinfold assessment? a. After pinching the skin and fat, the calipers are vertically applied to the fat fold. b. The skin and fat on the front of the patient’s arm are gently pinched, and then the calipers are applied. c. After applying the calipers, the nurse waits 3 seconds before taking a reading. After repeating the procedure three times, an average is recorded. d. The patient is instructed to stand with his or her back to the examiner and arms folded across the chest. The skin on the forearm is pinched. C While holding the skinfold, the lever of the calipers is released. The nurse waits 3 seconds and then takes a reading. This procedure should be repeated three times, and an average of the three skinfold measurements is then recorded. 18. In teaching a patient how to determine total body fat at home, the nurse includes instructions to obtain measurements of: a. Height and weight. b. Frame size and weight. c. Waist and hip circumferences. d. Mid-upper arm circumference and arm span. A Body mass index, calculated by using height and weight measurements, is a practical marker of optimal weight for height and an indicator of obesity. The other options are not correct. 19. The nurse is evaluating patients for obesity-related diseases by calculating the waist-to-hip ratios. Which one of these patients would be at increased risk? a. 29-year-old woman whose waist measures 33 inches and hips measure 36 inches b. 32-year-old man whose waist measures 34 inches and hips measure 36 inches c. 38-year-old man whose waist measures 35 inches and hips measure 38 inches d. 46-year-old woman whose waist measures 30 inches and hips measure 38 inches A The waist-to-hip ratio assesses body fat distribution as an indicator of health risk. A waist-to-hip ratio of 1.0 or greater in men or 0.8 or greater in women is indicative of android (upper body obesity) and increasing risk for obesity-related disease and early death. The 29-year-old woman has a waist-to-hip ratio of 0.92, which is greater than 0.8. The 32-year-old man has a waist-to-hip ratio of 0.94; the 38-year-old man has a waist-to-hip ratio of 0.92; the 46-year-old woman has a waist-to-hip ratio of 0.78. 20. A 50-year-old woman with elevated total cholesterol and triglyceride levels is visiting the clinic to find out about her laboratory results. What would be important for the nurse to include in patient teaching in relation to these tests? a. The risks of undernutrition should be included. b. Offer methods to reduce the stress in her life. c. Provide information regarding a diet low in saturated fat. d. This condition is hereditary; she can do nothing to change the levels. C The patient with elevated cholesterol and triglyceride levels should be taught about eating a healthy diet that limits the intake of foods high in saturated fats or trans fats. Reducing dietary fats is part of the treatment for this condition. The other responses are not pertinent to her condition. 21. In performing an assessment on a 49-year-old woman who has imbalanced nutrition as a result of dysphagia, which data would the nurse expect to find? a. Increase in hair growth b. Inadequate nutrient food intake c. Weight 10% to 20% over ideal d. Sore, inflamed buccal cavity B Dysphagia, or impaired swallowing, interferes with adequate nutrient intake. 22. A 21-year-old woman has been on a low-protein liquid diet for the past 2 months. She has had adequate intake of calories and appears well nourished. After further assessment, what would the nurse expect to find? a. Poor skin turgor b. Decreased serum albumin c. Increased lymphocyte count d. Triceps skinfold less than standard B Kwashiorkor (protein malnutrition) is due to diets that may be high in calories but contain little or no protein (e.g., low-protein liquid diets, fad diets, and long-term use of dextrose-containing intravenous fluids). The serum albumin would be less than 3.5 g/dL. 23. The nurse is performing a nutritional assessment on an 80-year-old patient. The nurse knows that physiologic changes can directly affect the nutritional status of the older adult and include: a. Slowed gastrointestinal motility. b. Hyperstimulation of the salivary glands. c. Increased sensitivity to spicy and aromatic foods. d. Decreased gastrointestinal absorption causing esophageal reflux. A Normal physiologic changes in aging adults that affect nutritional status include slowed gastrointestinal motility, decreased gastrointestinal absorption, diminished olfactory and taste sensitivity, decreased saliva production, decreased visual acuity, and poor dentition. 24. Which of these interventions is most appropriate when the nurse is planning nutritional interventions for a healthy, active 74-year-old woman? a. Decreasing the amount of carbohydrates to prevent lean muscle catabolism b. Increasing the amount of soy and tofu in her diet to promote bone growth and reverse osteoporosis c. Decreasing the number of calories she is eating because of the decrease in energy requirements from the loss of lean body mass d. Increasing the number of calories she is eating because of the increased energy needs of the older adult C Important nutritional features of the older years are a decrease in energy requirements as a result of loss of lean body mass, the most metabolically active tissue, and an increase in fat mass. 25. A 16-year-old girl is being seen at the clinic for gastrointestinal complaints and weight loss. The nurse determines that many of her complaints may be related to erratic eating patterns, eating predominantly fast foods, and high caffeine intake. In this situation, which is most appropriate when collecting current dietary intake information? a. Scheduling a time for direct observation of the adolescent during meals b. Asking the patient for a 24-hour diet recall, and assuming it to be reflective of a typical day for her c. Having the patient complete a food diary for 3 days, including 2 weekdays and 1 weekend day d. Using the food frequency questionnaire to identify the amount of intake of specific foods C Food diaries require the individual to write down everything consumed for a certain time period. Because of the erratic eating patterns of this individual, assessing dietary intake over a few days would produce more accurate information regarding eating patterns. Direct observation is best used with young children or older adults. 26. The nurse is preparing to measure fat and lean body mass and bone mineral density. Which tool is appropriate? a. Measuring tape b. Skinfold calipers c. Bioelectrical impedance analysis (BIA) d. Dual-energy x-ray absorptiometry (DEXA) : D DEXA measures both bone mineral density and fat and lean body mass. BIA measures fat and lean body mass but not bone mineral density. A measuring tape measures distance or length, and skinfold calipers are used to determine skinfold thickness. 27. Which of these conditions is due to an inadequate intake of both protein and calories? a. Obesity b. Bulimia c. Marasmus d. Kwashiorkor C Marasmus, protein-calorie malnutrition, is due to an inadequate intake of protein and calories or prolonged starvation. Obesity is due to caloric excess; bulimia is an eating disorder. Kwashiorkor is protein malnutrition. 28. During an assessment of a patient who has been homeless for several years, the nurse notices that his tongue is magenta in color, which is an indication of a deficiency in what mineral and/or vitamin? a. Iron b. Riboflavin c. Vitamin D and calcium d. Vitamin C B Magenta tongue is a sign of riboflavin deficiency. In contrast, a pale tongue is probably attributable to iron deficiency. Vitamin D and calcium deficiencies cause osteomalacia in adults, and a vitamin C deficiency causes scorbutic gums. 29. A 50-year-old patient has been brought to the emergency department after a housemate found that the patient could not get out of bed alone. He has lived in a group home for years but for several months has not participated in the activities and has stayed in his room. The nurse assesses for signs of undernutrition, and an x-ray study reveals that he has osteomalacia, which is a deficiency of: a. Iron. b. Riboflavin. c. Vitamin D and calcium. d. Vitamin C. C Osteomalacia results from a deficiency of vitamin D and calcium in adults. Iron deficiency would result in anemia, riboflavin deficiency would result in magenta tongue, and vitamin C deficiency would result in scurvy. 30. An older adult patient in a nursing home has been receiving tube feedings for several months. During an oral examination, the nurse notes that patient’s gums are swollen, ulcerated, and bleeding in some areas. The nurse suspects that the patient has what condition? a. Rickets b. Vitamin A deficiency c. Linoleic-acid deficiency d. Vitamin C deficiency D Vitamin C deficiency causes swollen, ulcerated, and bleeding gums, known as scorbutic gums. Rickets is a condition related to vitamin D and calcium deficiencies in infants and children. Linoleic-acid deficiency causes eczematous skin. Vitamin A deficiency causes Bitot spots and visual problems. 31. The nurse is assessing the body weight as a percentage of ideal body weight on an adolescent patient who was admitted for suspected anorexia nervosa. The patient’s usual weight was 125 pounds, but today she weighs 98 pounds. The nurse calculates the patient’s ideal body weight and concludes that the patient is: a. Experiencing mild malnutrition. b. Experiencing moderate malnutrition. c. Experiencing severe malnutrition. d. Still within expected parameters with her current weight. B By dividing her current weight by her usual weight and then multiplying by 100, a percentage of 78.4% is obtained, which means that her current weight is 78.4% of her ideal body weight. A current weight of 80% to 90% of ideal weight suggests mild malnutrition; a current weight of 70% to 80% of ideal weight suggests moderate malnutrition; a current weight of less than 70% of ideal weight suggests severe malnutrition. MULTIPLE RESPONSE 1. The nurse is assessing a patient who is obese for signs of metabolic syndrome. This condition is diagnosed when three or more certain risk factors are present. Which of these assessment findings are risk factors for metabolic syndrome? Select all that apply. a. Fasting plasma glucose level less than 100 mg/dL b. Fasting plasma glucose level greater than or equal to 110 mg/dL c. Blood pressure reading of 140/90 mm Hg d. Blood pressure reading of 110/80 mm Hg e. Triglyceride level of 120 mg/dL B, C Metabolic syndrome is diagnosed when three or more of the following risk factors are present: (1) fasting plasma glucose level greater than or equal to 100 mg/dL; (2) blood pressure greater than or equal to 130/85 mm Hg; (3) waist circumference greater than or equal to 40 inches for men and 35 inches for women; (4) high-density lipoprotein cholesterol less than 40 in men and less than 50 in women; and (5) triglyceride levels greater than or equal to 150 mg/dL (ATP III, 2001). SHORT ANSWER 1. A patient has been unable to eat solid food for 2 weeks and is in the clinic today complaining of weakness, tiredness, and hair loss. The patient states that her usual weight is 175 pounds, but today she weighs 161 pounds. What is her recent weight change percentage? To calculate recent weight change percentage, use this formula: Usual weight – current weight 100 usual weight 8% 175 – 161 = 14 pounds 14 ÷ 175 = 0.08 0.08  100 = 8% CHAPTER 12 1. The nurse educator is preparing an education module for the nursing staff on the epidermal layer of skin. Which of these statements would be included in the module? The epidermis is: a. Highly vascular. b. Thick and tough. c. Thin and nonstratified. d. Replaced every 4 weeks. D The epidermis is thin yet tough, replaced every 4 weeks, avascular, and stratified into several zones. 2. The nurse educator is preparing an education module for the nursing staff on the dermis layer of skin. Which of these statements would be included in the module? The dermis: a. Contains mostly fat cells. b. Consists mostly of keratin. c. Is replaced every 4 weeks. d. Contains sensory receptors. D The dermis consists mostly of collagen, has resilient elastic tissue that allows the skin to stretch, and contains nerves, sensory receptors, blood vessels, and lymphatic vessels. It is not replaced every 4 weeks. 3. The nurse is examining a patient who tells the nurse, “I sure sweat a lot, especially on my face and feet but it doesn’t have an odor.” The nurse knows that this condition could be related to: a. Eccrine glands. b. Apocrine glands. c. Disorder of the stratum corneum. d. Disorder of the stratum germinativum. A The eccrine glands are coiled tubules that directly open onto the skin surface and produce a dilute saline solution called sweat. Apocrine glands are primarily located in the axillae, anogenital area, nipples, and naval area and mix with bacterial flora to produce the characteristic musky body odor. The patient’s statement is not related to disorders of the stratum corneum or the stratum germinativum. 4. A newborn infant is in the clinic for a well-baby checkup. The nurse observes the infant for the possibility of fluid loss because of which of these factors? a. Subcutaneous fat deposits are high in the newborn. b. Sebaceous glands are overproductive in the newborn. c. The newborn’s skin is more permeable than that of the adult. d. The amount of vernixcaseosa dramatically rises in the newborn. C The newborn’s skin is thin, smooth, and elastic and is relatively more permeable than that of the adult; consequently, the infant is at greater risk for fluid loss. The subcutaneous layer in the infant is inefficient, not thick, and the sebaceous glands are present but decrease in size and production. Vernixcaseosa is not produced after birth. 5. The nurse is bathing an 80-year-old man and notices that his skin is wrinkled, thin, lax, and dry. This finding would be related to which factor in the older adult? a. Increased vascularity of the skin b. Increased numbers of sweat and sebaceous glands c. An increase in elastin and a decrease in subcutaneous fat d. An increased loss of elastin and a decrease in subcutaneous fat D An accumulation of factors place the aging person at risk for skin disease and breakdown: the thinning of the skin, a decrease in vascularity and nutrients, the loss of protective cushioning of the subcutaneous layer, a lifetime of environmental trauma to skin, the social changes of aging, a increasingly sedentary lifestyle, and the chance of immobility. 6. During the aging process, the hair can look gray or white and begin to feel thin and fine. The nurse knows that this occurs because of a decrease in the number of functioning: a. Metrocytes. b. Fungacytes. c. Phagocytes. d. Melanocytes. ANS: D In the aging hair matrix, the number of functioning melanocytes decreases; as a result, the hair looks gray or white and feels thin and fine. The other options are not correct. 7. During an examination, the nurse finds that a patient has excessive dryness of the skin. The best term to describe this condition is: a. Xerosis. b. Pruritus. c. Alopecia. d. Seborrhea. A Xerosis is the term used to describe skin that is excessively dry. Pruritus refers to itching, alopecia refers to hair loss, and seborrhea refers to oily skin. 8. A 22-year-old woman comes to the clinic because of severe sunburn and states, “I was out in the sun for just a couple of minutes.” The nurse begins a medication review with her, paying special attention to which medication class? a. Nonsteroidalantiinflammatory drugs for pain b. Tetracyclines for acne c. Proton pump inhibitors for heartburn d. Thyroid replacement hormone for hypothyroidism B Drugs that may increase sunlight sensitivity and give a burn response include sulfonamides, thiazide diuretics, oral hypoglycemic agents, and tetracycline. 9. A woman is leaving on a trip to Hawaii and has come in for a checkup. During the examination the nurse learns that she has diabetes and takes oral hypoglycemic agents. The patient needs to be concerned about which possible effect of her medications? a. Increased possibility of bruising b. Skin sensitivity as a result of exposure to salt water c. Lack of availability of glucose-monitoring supplies d. Importance of sunscreen and avoiding direct sunlight D Drugs that may increase sunlight sensitivity and give a burn response include sulfonamides, thiazide diuretics, oral hypoglycemic agents, and tetracycline. 10. A 13-year-old girl is interested in obtaining information about the cause of her acne. The nurse should share with her that acne: a. Is contagious. b. Has no known cause. c. Is caused by increased sebum production. d. Has been found to be related to poor hygiene. C Approximately 90% of males and 80% of females will develop acne; causes are increased sebum production and epithelial cells that do not desquamate normally. 11. A 75-year-old woman who has a history of diabetes and peripheral vascular disease has been trying to remove a corn on the bottom of her foot with a pair of scissors. The nurse will encourage her to stop trying to remove the corn with scissors because: a. The woman could be at increased risk for infection and lesions because of her chronic disease. b. With her diabetes, she has increased circulation to her foot, and it could cause severe bleeding. c. She is 75 years old and is unable to see; consequently, she places herself at greater risk for self-injury with the scissors. d. With her peripheral vascular disease, her range of motion is limited and she may not be able to reach the corn safely. A A personal history of diabetes and peripheral vascular disease increases a person’s risk for skin lesions in the feet or ankles. The patient needs to seek a professional for assistance with corn removal. 12. The nurse keeps in mind that a thorough skin assessment is extremely important because the skin holds information about a person’s: a. Support systems. b. Circulatory status. c. Socioeconomic status. d. Psychological wellness. B The skin holds information about the body’s circulation, nutritional status, and signs of systemic diseases, as well as topical data on the integumentary system itself. 13. A patient comes in for a physical examination and complains of “freezing to death” while waiting for her examination. The nurse notes that her skin is pale and cool and attributes this finding to: a. Venous pooling. b. Peripheral vasodilation. c. Peripheral vasoconstriction. d. Decreased arterial perfusion. C A chilly or air-conditioned environment causes vasoconstriction, which results in false pallor and coolness (see Table 12-1). 14. A patient comes to the clinic and tells the nurse that he has been confined to his recliner chair for approximately 3 days with his feet down and he asks the nurse to evaluate his feet. During the assessment, the nurse might expect to find: a. Pallor b. Coolness c. Distended veins d. Prolonged capillary filling time C Keeping the feet in a dependent position causes venous pooling, resulting in redness, warmth, and distended veins. Prolonged elevation would cause pallor and coolness. Immobilization or prolonged inactivity would cause prolonged capillary filling time (see Table 12-1). 15. A patient is especially worried about an area of skin on her feet that has turned white. The health care provider has told her that her condition is vitiligo. The nurse explains to her that vitiligo is: a. Caused by an excess of melanin pigment b. Caused by an excess of apocrine glands in her feet c. Caused by the complete absence of melanin pigment d. Related to impetigo and can be treated with an ointment C Vitiligo is the complete absence of melanin pigment in patchy areas of white or light skin on the face, neck, hands, feet, body folds, and around orifices—otherwise, the depigmented skin is normal. 16. A patient tells the nurse that he has noticed that one of his moles has started to burn and bleed. When assessing his skin, the nurse pays special attention to the danger signs for pigmented lesions and is concerned with which additional finding? a. Color variation b. Border regularity c. Symmetry of lesions d. Diameter of less than 6 mm A Abnormal characteristics of pigmented lesions are summarized in the mnemonic ABCD: asymmetry of pigmented lesion, border irregularity, color variation, and diameter greater than 6 mm. 17. A patient comes to the clinic and states that he has noticed that his skin is redder than normal. The nurse understands that this condition is due to hyperemia and knows that it can be caused by: a. Decreased amounts of bilirubin in the blood b. Excess blood in the underlying blood vessels c. Decreased perfusion to the surrounding tissues d. Excess blood in the dilated superficial capillaries D Erythema is an intense redness of the skin caused by excess blood (hyperemia) in the dilated superficial capillaries. 18. During a skin assessment, the nurse notices that a Mexican-American patient has skin that is yellowish-brown; however, the skin on the hard and soft palate is pink and the patient’s scleras are not yellow. From this finding, the nurse could probably rule out: a. Pallor b. Jaundice c. Cyanosis d. Iron deficiency B Jaundice is exhibited by a yellow color, which indicates rising levels of bilirubin in the blood. Jaundice is first noticed in the junction of the hard and soft palate in the mouth and in the scleras. 19. A black patient is in the intensive care unit because of impending shock after an accident. The nurse expects to find what characteristics in this patient’s skin? a. Ruddy blue. b. Generalized pallor. c. Ashen, gray, or dull. d. Patchy areas of pallor. C Pallor attributable to shock, with decreased perfusion and vasoconstriction, in black-skinned people will cause the skin to appear ashen, gray, or dull (see Table 12-2). Client Needs: Physiologic Integrity: Physiologic Adaptation 20. An older adult woman is brought to the emergency department after being found lying on the kitchen floor for 2 days; she is extremely dehydrated. What would the nurse expect to see during the examination? a. Smooth mucous membranes and lips b. Dry mucous membranes and cracked lips c. Pale mucous membranes d. White patches on the mucous membranes B With dehydration, mucous membranes appear dry and the lips look parched and cracked. The other responses are not found in dehydration. 21. A 42-year-old woman complains that she has noticed several small, slightly raised, bright red dots on her chest. On examination, the nurse expects that the spots are probably: a. Anasarca. b. Scleroderma. c. Senile angiomas. d. Latent myeloma. C Cherry (senile) angiomas are small, smooth, slightly raised bright red dots that commonly appear on the trunk of adults over 30 years old. 22. A 65-year-old man with emphysema and bronchitis has come to the clinic for a follow-up appointment. On assessment, the nurse might expect to see which finding? a. Anasarca b. Scleroderma c. Pedal erythema d. Clubbing of the nails D Clubbing of the nails occurs with congenital cyanotic heart disease and neoplastic and pulmonary diseases. The other responses are assessment findings not associated with pulmonary diseases. 23. A newborn infant has Down syndrome. During the skin assessment, the nurse notices a transient mottling in the trunk and extremities in response to the cool temperature in the examination room. The infant’s mother also notices the mottling and asks what it is. The nurse knows that this mottling is called: a. Café au lait. b. Carotenemia. c. Acrocyanosis. d. Cutis marmorata. D Persistent or pronounced cutis marmorata occurs with infants born with Down syndrome or those born prematurely and is a transient mottling in the trunk and extremities in response to cool room temperatures. A café au lait spot is a large round or oval patch of light-brown pigmentation. Carotenemia produces a yellow-orange color in light-skinned persons. Acrocyanosis is a bluish color around the lips, hands and fingernails, and feet and toenails. 24. A 35-year-old pregnant woman comes to the clinic for a monthly appointment. During the assessment, the nurse notices that she has a brown patch of hyperpigmentation on her face. The nurse continues the skin assessment aware that another finding may be: a. Keratoses. b. Xerosis. c. Chloasma. d. Acrochordons. C In pregnancy, skin changes can include striae, lineanigra (a brownish-black line down the midline), chloasma (brown patches of hyperpigmentation), and vascular spiders. Keratoses are raised, thickened areas of pigmentation that look crusted, scaly, and warty. Xerosis is dry skin. Acrochordons, or skin tags, occur more often in the aging adult. 25. A man has come in to the clinic for a skin assessment because he is worried he might have skin cancer. During the skin assessment the nurse notices several areas of pigmentation that look greasy, dark, and “stuck on” his skin. Which is the best prediction? a. Senile lentigines, which do not become cancerous b. Actinic keratoses, which are precursors to basal cell carcinoma c. Acrochordons, which are precursors to squamous cell carcinoma d. Seborrheickeratoses, which do not become cancerous D Seborrheickeratoses appear like dark, greasy, “stuck-on” lesions that primarily develop on the trunk. These lesions do not become cancerous. Senile lentigines are commonly called liver spots and are not precancerous. Actinic (senile or solar) keratoses are lesions that are red-tan scaly plaques that increase over the years to become raised and roughened. They may have a silvery-white scale adherent to the plaque. They occur on sun-exposed surfaces and are directly related to sun exposure. They are premalignant and may develop into squamous cell carcinoma. Acrochordons are skin tags and are not precancerous. 26. A 70-year-old woman who loves to garden has small, flat, brown macules over her arms and hands. She asks, “What causes these liver spots?” The nurse tells her, “They are: a. “Signs of decreased hematocrit related to anemia.” b. “Due to the destruction of melanin in your skin from exposure to the sun.” c. “Clusters of melanocytes that appear after extensive sun exposure.” d. “Areas of hyperpigmentation related to decreased perfusion and vasoconstriction.” C Liver spots, or senile lentigines, are clusters of melanocytes that appear on the forearms and dorsa of the hands after extensive sun exposure. The other responses are not correct. 27. The nurse notices that a patient has a solid, elevated, circumscribed lesion that is less than 1 cm in diameter. When documenting this finding, the nurse reports this as a: a. Bulla. b. Wheal. c. Nodule. d. Papule. D A papule is something one can feel, is solid, elevated, circumscribed, less than 1 cm in diameter, and is due to superficial thickening in the epidermis. A bulla is larger than 1 cm, superficial, and thin walled. A wheal is superficial, raised, transient, erythematous, and irregular in shape attributable to edema. A nodule is solid, elevated, hard or soft, and larger than 1 cm. 28. The nurse just noted from the medical record that the patient has a lesion that is confluent in nature. On examination, the nurse expects to find: a. Lesions that run together. b. Annular lesions that have grown together. c. Lesions arranged in a line along a nerve route. d. Lesions that are grouped or clustered together. A Confluent lesions (as with urticaria [hives]) run together. Grouped lesions are clustered together. Annular lesions are circular in nature. Zosteriform lesions are arranged along a nerve route. 29. A patient has had a “terrible itch” for several months that he has been continuously scratching. On examination, the nurse might expect to find: a. A keloid. b. A fissure. c. Keratosis. d. Lichenification. D Lichenification results from prolonged, intense scratching that eventually thickens the skin and produces tightly packed sets of papules. A keloid is a hypertrophic scar. A fissure is a linear crack with abrupt edges, which extends into the dermis; it can be dry or moist. Keratoses are lesions that are raised, thickened areas of pigmentation that appear crusted, scaly, and warty. 30. A physician has diagnosed a patient with purpura. After leaving the room, a nursing student asks the nurse what the physician saw that led to that diagnosis. The nurse should say, “The physician is referring to the: a. “Blue dilation of blood vessels in a star-shaped linear pattern on the legs.” b. “Fiery red, star-shaped marking on the cheek that has a solid circular center.” c. “Confluent and extensive patch of petechiae and ecchymoses on the feet.” d. “Tiny areas of hemorrhage that are less than 2 mm, round, discrete, and dark red in color.” C Purpura is a confluent and extensive patch of petechiae and ecchymoses and a flat macular hemorrhage observed in generalized disorders such as thrombocytopenia and scurvy. The blue dilation of blood vessels in a star-shaped linear pattern on the legs describes a venous lake. The fiery red, star-shaped marking on the cheek that has a solid circular center describes a spider or star angioma. The tiny areas of hemorrhage that are less than 2 mm, round, discrete, and dark red in color describespetechiae. 31. A mother has noticed that her son, who has been to a new babysitter, has some blisters and scabs on his face and buttocks. On examination, the nurse notices moist, thin-roofed vesicles with a thin erythematous base and suspects: a. Eczema. b. Impetigo. c. Herpes zoster. d. Diaper dermatitis. B Impetigo is moist, thin-roofed vesicles with a thin erythematous base and is a contagious bacterial infection of the skin and most common in infants and children. Eczema is characterized by erythematous papules and vesicles with weeping, oozing, and crusts. Herpes zoster (i.e., chickenpox or varicella) is characterized by small, tight vesicles that are shiny with an erythematous base. Diaper dermatitis is characterized by red, moist maculopapular patches with poorly defined borders. 32. The nurse notices that a school-aged child has bluish-white, red-based spots in her mouth that are elevated approximately 1 to 3 mm. What other signs would the nurse expect to find in this patient? a. Pink, papular rash on the face and neck b. Pruritic vesicles over her trunk and neck c. Hyperpigmentation on the chest, abdomen, and back of the arms d. Red-purple, maculopapular, blotchy rash behind the ears and on the face D With measles (rubeola), the examiner assesses a red-purple, blotchy rash on the third or fourth day of illness that appears first behind the ears, spreads over the face, and then over the neck, trunk, arms, and legs. The rash appears coppery and does not blanch. The bluish-white, red-based spots in the mouth are known as Koplik spots. 33. The nurse is assessing the skin of a patient who has acquired immunodeficiency syndrome (AIDS) and notices multiple patchlike lesions on the temple and beard area that are faint pink in color. The nurse recognizes these lesions as: a. Measles (rubeola). b. Kaposi’s sarcoma. c. Angiomas. d. Herpes zoster. B Kaposi’s sarcoma is a vascular tumor that, in the early stages, appears as multiple, patchlike, faint pink lesions over the patient’s temple and beard areas. Measles is characterized by a red-purple maculopapular blotchy rash that appears on the third or fourth day of illness. The rash is first observed behind the ears, spreads over the face, and then spreads over the neck, trunk, arms, and legs. Cherry (senile) angiomas are small (1 to 5 mm), smooth, slightly raised bright red dots that commonly appear on the trunk in all adults over 30 years old. Herpes zoster causes vesicles up to 1 cm in size that are elevated with a cavity containing clear fluid. 34. A 45-year-old farmer comes in for a skin evaluation and complains of hair loss on his head. His hair seems to be breaking off in patches, and he notices some scaling on his head. The nurse begins the examination suspecting: a. Tineacapitis. b. Folliculitis. c. Toxic alopecia. d. Seborrheic dermatitis. A Tineacapitis is rounded patchy hair loss on the scalp, leaving broken-off hairs, pustules, and scales on the skin, and is caused by a fungal infection. Lesions are fluorescent under a Wood light and are usually observed in children and farmers; tineacapitis is highly contagious. (See Table 12-12, Abnormal Conditions of Hair, for descriptions of the other terms.) 35. A mother brings her child into the clinic for an examination of the scalp and hair. She states that the child has developed irregularly shaped patches with broken-off, stublike hair in some places; she is worried that this condition could be some form of premature baldness. The nurse tells her that it is: a. Folliculitis that can be treated with an antibiotic. b. Traumatic alopecia that can be treated with antifungal medications. c. Tineacapitis that is highly contagious and needs immediate attention. d. Trichotillomania; her child probably has a habit of absentmindedly twirling her hair. D Trichotillomania, self-induced hair loss, is usually due to habit. It forms irregularly shaped patches with broken-off, stublike hairs of varying lengths. A person is never completely bald. It occurs as a child absentmindedly rubs or twirls the area while falling asleep, reading, or watching television. (See Table 12-12, Abnormal Conditions of Hair, for descriptions of the other terms.) 36. The nurse has discovered decreased skin turgor in a patient and knows that this finding is expected in which condition? a. Severe obesity b. Childhood growth spurts c. Severe dehydration d. Connective tissue disorders such as scleroderma C Decreased skin turgor is associated with severe dehydration or extreme weight loss. 37. While performing an assessment of a 65-year-old man with a history of hypertension and coronary artery disease, the nurse notices the presence of bilateral pitting edema in the lower legs. The skin is puffy and tight but normal in color. No increased redness or tenderness is observed over his lower legs, and the peripheral pulses are equal and strong. In this situation, the nurse suspects that the likely cause of the edema is which condition? a. Heart failure b. Venous thrombosis c. Local inflammation d. Blockage of lymphatic drainage A Bilateral edema or edema that is generalized over the entire body is caused by a central problem such as heart failure or kidney failure. Unilateral edema usually has a local or peripheral cause. 38. A 40-year-old woman reports a change in mole size, accompanied by color changes, itching, burning, and bleeding over the past month. She has a dark complexion and has no family history of skin cancer, but she has had many blistering sunburns in the past. The nurse would: a. Tell the patient to watch the lesion and report back in 2 months. b. Refer the patient because of the suggestion of melanoma on the basis of her symptoms. c. Ask additional questions regarding environmental irritants that may have caused this condition. d. Tell the patient that these signs suggest a compound nevus, which is very common in young to middle-aged adults. B The ABCD danger signs of melanoma are asymmetry, border irregularity, color variation, and diameter. In addition, individuals may report a change in size, the development of itching, burning, and bleeding, or a new-pigmented lesion. Any one of these signs raises the suggestion of melanoma and warrants immediate referral. 39. The nurse is assessing for clubbing of the fingernails and expects to find: a. Nail bases that are firm and slightly tender. b. Curved nails with a convex profile and ridges across the nails. c. Nail bases that feel spongy with an angle of the nail base of 150 degrees. d. Nail bases with an angle of 180 degrees or greater and nail bases that feel spongy. D The normal nail is firm at its base and has an angle of 160 degrees. In clubbing, the angle straightens to 180 degrees or greater and the nail base feels spongy. 40. The nurse is assessing a patient who has liver disease for jaundice. Which of these assessment findings is indicative of true jaundice? a. Yellow patches in the outer sclera b. Yellow color of the sclera that extends up to the iris c. Skin that appears yellow when examined under low light d. Yellow deposits on the palms and soles of the feet where jaundice first appears B The yellow sclera of jaundice extends up to the edge of the iris. Calluses on the palms and soles of the feet often appear yellow but are not classified as jaundice. Scleral jaundice should not be confused with the normal yellow subconjunctival fatty deposits that are common in the outer sclera of dark-skinned persons. 41. The nurse is assessing for inflammation in a dark-skinned person. Which technique is the best? a. Assessing the skin for cyanosis and swelling b. Assessing the oral mucosa for generalized erythema c. Palpating the skin for edema and increased warmth d. Palpating for tenderness and local areas of ecchymosis C Because inflammation cannot be seen in dark-skinned persons, palpating the skin for increased warmth, for taut or tightly pulled surfaces that may be indicative of edema, and for a hardening of deep tissues or blood vessels is often necessary. 42. A few days after a summer hiking trip, a 25-year-old man comes to the clinic with a rash. On examination, the nurse notes that the rash is red, macular, with a bull’s eye pattern across his midriff and behind his knees. The nurse suspects: a. Rubeola. b. Lyme disease. c. Allergy to mosquito bites. d. Rocky Mountain spotted fever. B Lyme disease occurs in people who spend time outdoors in May through September. The first disease state exhibits the distinctive bull’s eye and a red macular or papular rash that radiates from the site of the tick bite with some central clearing. The rash spreads 5 cm or larger, and is usually in the axilla, midriff, inguinal, or behind the knee, with regional lymphadenopathy. 43. A 52-year-old woman has a papule on her nose that has rounded, pearly borders and a central red ulcer. She said she first noticed it several months ago and that it has slowly grown larger. The nurse suspects which condition? a. Acne b. Basal cell carcinoma c. Melanoma d. Squamous cell carcinoma B Basal cell carcinoma usually starts as a skin-colored papule that develops rounded, pearly borders with a central red ulcer. It is the most common form of skin cancer and grows slowly. This description does not fit acne lesions. (See Table 12-11 for descriptions of melanoma and squamous cell carcinoma.) 44. A father brings in his 2-month-old infant to the clinic because the infant has had diarrhea for the last 24 hours. He says his baby has not been able to keep any formula down and that the diarrhea has been at least every 2 hours. The nurse suspects dehydration. The nurse should test skin mobility and turgor over the infant’s: a. Sternum. b. Forehead. c. Forearms. d. Abdomen. D Mobility and turgor are tested over the abdomen in an infant. Poor turgor, or tenting, indicates dehydration or malnutrition. The other sites are not appropriate for checking skin turgor in an infant. 45. A semiconscious woman is brought to the emergency department after she was found on the floor in her kitchen. Her face, nail beds, lips, and oral mucosa are a bright cherry-red color. The nurse suspects that this coloring is due to: a. Polycythemia. b. Carbon monoxide poisoning. c. Carotenemia. d. Uremia. B A bright cherry-red coloring in the face, upper torso, nail beds, lips, and oral mucosa appears in cases of carbon monoxide poisoning. 46. A patient has been admitted for severe psoriasis. The nurse expects to see what finding in the patient’s fingernails? a. Splinter hemorrhages b. Paronychia c. Pitting d. Beau lines C Sharply defined pitting and crumbling of the nails, each with distal detachment characterize pitting nails and are associated with psoriasis. (See Table 12-13 for descriptions of the other terms.) MULTIPLE RESPONSE 1. The nurse is preparing for a certification course in skin care and needs to be familiar with the various lesions that may be identified on assessment of the skin. Which of the following definitions are correct? Select all that apply. a. Petechiae: Tiny punctate hemorrhages, 1 to 3 mm, round and discrete, dark red, purple, or brown in color b. Bulla: Elevated, circumscribed lesion filled with turbid fluid (pus) c. Papule: Hypertrophic scar d. Vesicle: Known as a friction blister e. Nodule: Solid, elevated, and hard or soft growth that is larger than 1 cm A, D, E A pustule is an elevated, circumscribed lesion filled with turbid fluid (pus). A hypertrophic scar is a keloid. A bulla is larger than 1 cm and contains clear fluid. A papule is solid and elevated but measures less than 1 cm. 2. A patient has been admitted to a hospital after the staff in the nursing home noticed a pressure ulcer in his sacral area. The nurse examines the pressure ulcer and determines that it is a stage II ulcer. Which of these findings are characteristic of a stage II pressure ulcer? Select all that apply. a. Intact skin appears red but is not broken. b. Partial thickness skin erosion is observed with a loss of epidermis or dermis. c. Ulcer extends into the subcutaneous tissue. d. Localized redness in light skin will blanch with fingertip pressure. e. Open blister areas have a red-pink wound bed. f. Patches of eschar cover parts of the wound. B, E Stage I pressure ulcers have intact skin that appears red but is not broken, and localized redness in intact skin will blanche with fingertip pressure. Stage II pressure ulcers have partial thickness skin erosion with a loss of epidermis or also the dermis; open blisters have a red-pink wound bed. Stage III pressure ulcers are full thickness, extending into the subcutaneous tissue; subcutaneous fat may be seen but not muscle, bone, or tendon. Stage IV pressure ulcers involve all skin layers and extend into supporting tissue, exposing muscle, bone, and tendon. Slough (stringy matter attached to the wound bed) or eschar (black or brown necrotic tissue) may be present. CHAPTER 13 1. A physician tells the nurse that a patient’s vertebra prominens is tender and asks the nurse to reevaluate the area in 1 hour. The area of the body the nurse will assess is: a. Just above the diaphragm. b. Just lateral to the knee cap. c. At the level of the C7 vertebra. d. At the level of the T11 vertebra. C The C7 vertebra has a long spinous process, called the vertebra prominens, which is palpable when the head is flexed. 2. A mother brings her 2-month-old daughter in for an examination and says, “My daughter rolled over against the wall, and now I have noticed that she has this spot that is soft on the top of her head. Is something terribly wrong?” The nurse’s best response would be: a. “Perhaps that could be a result of your dietary intake during pregnancy.” b. “Your baby may have craniosynostosis, a disease of the sutures of the brain.” c. “That ‘soft spot’ may be an indication of cretinism or congenital hypothyroidism.” d. “That ‘soft spot’ is normal, and actually allows for growth of the brain during the first year of your baby’s life.” D Membrane-covered “soft spots” allow for growth of the brain during the first year of life. They gradually ossify; the triangular-shaped posterior fontanel is closed by 1 to 2 months, and the diamond-shaped anterior fontanel closes between 9 months and 2 years. 3. The nurse notices that a patient’s palpebral fissures are not symmetric. On examination, the nurse may find that damage has occurred to which cranial nerve (CN)? a. III b. V c. VII d. VIII C Facial muscles are mediated by CN VII; asymmetry of palpebral fissures may be attributable to damage to CN VII (Bell palsy). 4. A patient is unable to differentiate between sharp and dull stimulation to both sides of her face. The nurse suspects: a. Bell palsy. b. Damage to the trigeminal nerve. c. Frostbite with resultant paresthesia to the cheeks. d. Scleroderma. B Facial sensations of pain or touch are mediated by CN V, which is the trigeminal nerve. Bell palsy is associated with CN VII damage. Frostbite and scleroderma are not associated with this problem. 5. When examining the face of a patient, the nurse is aware that the two pairs of salivary glands that are accessible to examination are the ___________ and ___________ glands. a. Occipital; submental b. Parotid; jugulodigastric c. Parotid; submandibular d. Submandibular; occipital C Two pairs of salivary glands accessible to examination on the face are the parotid glands, which are in the cheeks over the mandible, anterior to and below the ear; and the submandibular glands, which are beneath the mandible at the angle of the jaw. The parotid glands are normally nonpalpable. 6. A patient comes to the clinic complaining of neck and shoulder pain and is unable to turn her head. The nurse suspects damage to CN ______ and proceeds with the examination by _____________. a. XI; palpating the anterior and posterior triangles b. XI; asking the patient to shrug her shoulders against resistance c. XII; percussing the sternomastoid and submandibular neck muscles d. XII; assessing for a positive Romberg sign B The major neck muscles are the sternomastoid and the trapezius. They are innervated by CN XI, the spinal accessory. The innervated muscles assist with head rotation and head flexion, movement of the shoulders, and extension and turning of the head. 7. When examining a patient’s CN function, the nurse remembers that the muscles in the neck that are innervated by CN XI are the: a. Sternomastoid and trapezius. b. Spinal accessory and omohyoid. c. Trapezius and sternomandibular. d. Sternomandibular and spinal accessory. A The major neck muscles are the sternomastoid and the trapezius. They are innervated by CN XI, the spinal accessory. 8. A patient’s laboratory data reveal an elevated thyroxine (T4) level. The nurse would proceed with an examination of the _____ gland. a. Thyroid b. Parotid c. Adrenal d. Parathyroid A The thyroid gland is a highly vascular endocrine gland that secretes T4 and triiodothyronine (T3). The other glands do not secrete T4. 9. A patient says that she has recently noticed a lump in the front of her neck below her “Adam’s apple” that seems to be getting bigger. During the assessment, the finding that leads the nurse to suspect that this may not be a cancerous thyroid nodule is that the lump (nodule): a. Is tender. b. Is mobile and not hard. c. Disappears when the patient smiles. d. Is hard and fixed to the surrounding structures. B Painless, rapidly growing nodules may be cancerous, especially the appearance of a single nodule in a young person. However, cancerous nodules tend to be hard and fixed to surrounding structures, not mobile. 10. The nurse notices that a patient’s submental lymph nodes are enlarged. In an effort to identify the cause of the node enlargement, the nurse would assess the patient’s: a. Infraclavicular area. b. Supraclavicular area. c. Area distal to the enlarged node. d. Area proximal to the enlarged node. D When nodes are abnormal, the nurse should check the area into which they drain for the source of the problem. The area proximal (upstream) to the location of the abnormal node should be explored. 11. The nurse is aware that the four areas in the body where lymph nodes are accessible are the: a. Head, breasts, groin, and abdomen. b. Arms, breasts, inguinal area, and legs. c. Head and neck, arms, breasts, and axillae. d. Head and neck, arms, inguinal area, and axillae. D Nodes are located throughout the body, but they are accessible to examination only in four areas: head and neck, arms, inguinal region, and axillae. 12. A mother brings her newborn in for an assessment and asks, “Is there something wrong with my baby? His head seems so big.” Which statement is true regarding the relative proportions of the head and trunk of the newborn? a. At birth, the head is one fifth the total length. b. Head circumference should be greater than chest circumference at birth. c. The head size reaches 90% of its final size when the child is 3 years old. d. When the anterior fontanel closes at 2 months, the head will be more proportioned to the body. B The nurse recognizes that during the fetal period, head growth predominates. Head size is greater than chest circumference at birth, and the head size grows during childhood, reaching 90% of its final size when the child is age 6 years. 13. A patient, an 85-year-old woman, is complaining about the fact that the bones in her face have become more noticeable. What explanation should the nurse give her? a. Diets low in protein and high in carbohydrates may cause enhanced facial bones. b. Bones can become more noticeable if the person does not use a dermatologically approved moisturizer. c. More noticeable facial bones are probably due to a combination of factors related to aging, such as decreased elasticity, subcutaneous fat, and moisture in her skin. d. Facial skin becomes more elastic with age. This increased elasticity causes the skin to be more taught, drawing attention to the facial bones. C The facial bones and orbits appear more prominent in the aging adult, and the facial skin sags, which is attributable to decreased elasticity, decreased subcutaneous fat, and decreased moisture in the skin. 14. A patient reports excruciating headache pain on one side of his head, especially around his eye, forehead, and cheek that has lasted approximately to 2 hours, occurring once or twice each day. The nurse should suspect: a. Hypertension. b. Cluster headaches. c. Tension headaches. d. Migraine headaches. B Cluster headaches produce pain around the eye, temple, forehead, and cheek and are unilateral and always on the same side of the head. They are excruciating and occur once or twice per day and last to 2 hours each. 15. A patient complains that while studying for an examination he began to notice a severe headache in the frontotemporal area of his head that is throbbing and is somewhat relieved when he lies down. He tells the nurse that his mother also had these headaches. The nurse suspects that he may be suffering from: a. Hypertension. b. Cluster headaches. c. Tension headaches. d. Migraine headaches. D Migraine headaches tend to be supraorbital, retroorbital, or frontotemporal with a throbbing quality. They are severe in quality and are relieved by lying down. Migraines are associated with a family history of migraine headaches. 16. A 19-year-old college student is brought to the emergency department with a severe headache he describes as, “Like nothing I’ve ever had before.” His temperature is 40° C, and he has a stiff neck. The nurse looks for other signs and symptoms of which problem? a. Head injury b. Cluster headache c. Migraine headache d. Meningeal inflammation D The acute onset of neck stiffness and pain along with headache and fever occurs with meningeal inflammation. A severe headache in an adult or child who has never had it before is a red flag. Head injury and cluster or migraine headaches are not associated with a fever or stiff neck. 17. During a well-baby checkup, the nurse notices that a 1-week-old infant’s face looks small compared with his cranium, which seems enlarged. On further examination, the nurse also notices dilated scalp veins and downcast or “setting sun” eyes. The nurse suspects which condition? a. Craniotabes b. Microcephaly c. Hydrocephalus d. Caput succedaneum C Hydrocephalus occurs with the obstruction of drainage of cerebrospinal fluid that results in excessive accumulation, increasing intracranial pressure, and an enlargement of the head. The face looks small, compared with the enlarged cranium, and dilated scalp veins and downcast or “setting sun” eyes are noted. Craniotabes is a softening of the skull’s outer layer. Microcephaly is an abnormally small head. A caput succedaneum is edematous swelling and ecchymosis of the presenting part of the head caused by birth trauma. 18. The nurse needs to palpate the temporomandibular joint for crepitation. This joint is located just below the temporal artery and anterior to the: a. Hyoid bone. b. Vagus nerve. c. Tragus. d. Mandible. C The temporomandibular joint is just below the temporal artery and anterior to the tragus. 19. A patient has come in for an examination and states, “I have this spot in front of my ear lobe on my cheek that seems to be getting bigger and is tender. What do you think it is?” The nurse notes swelling below the angle of the jaw and suspects that it could be an inflammation of his: a. Thyroid gland. b. Parotid gland. c. Occipital lymph node. d. Submental lymph node. B Swelling of the parotid gland is evident below the angle of the jaw and is most visible when the head is extended. Painful inflammation occurs with mumps, and swelling also occurs with abscesses or tumors. Swelling occurs anterior to the lower ear lobe. 20. A male patient with a history of acquired immunodeficiency syndrome (AIDS) has come in for an examination and he states, “I think that I have the mumps.” The nurse would begin by examining the: a. Thyroid gland. b. Parotid gland. c. Cervical lymph nodes. d. Mouth and skin for lesions. B The parotid gland may become swollen with the onset of mumps, and parotid enlargement has been found with human immunodeficiency virus (HIV). 21. The nurse suspects that a patient has hyperthyroidism, and the laboratory data indicate that the patient’s T4 and T3 hormone levels are elevated. Which of these findings would the nurse most likely find on examination? a. Tachycardia b. Constipation c. Rapid dyspnea d. Atrophied nodular thyroid gland A T4 and T3 are thyroid hormones that stimulate the rate of cellular metabolism, resulting in tachycardia. With an enlarged thyroid gland as in hyperthyroidism, the nurse might expect to find diffuse enlargement (goiter) or a nodular lump but not an atrophied gland. Dyspnea and constipation are not findings associated with hyperthyroidism. 22. A visitor from Poland who does not speak English seems to be somewhat apprehensive about the nurse examining his neck. He would probably be more comfortable with the nurse examining his thyroid gland from: a. Behind with the nurse’s hands placed firmly around his neck. b. The side with the nurse’s eyes averted toward the ceiling and thumbs on his neck. c. The front with the nurse’s thumbs placed on either side of his trachea and his head tilted forward. d. The front with the nurse’s thumbs placed on either side of his trachea and his head tilted backward. C Examining this patient’s thyroid gland from the back may be unsettling for him. It would be best to examine his thyroid gland using the anterior approach, asking him to tip his head forward and to the right and then to the left. 23. A patient’s thyroid gland is enlarged, and the nurse is preparing to auscultate the thyroid gland for the presence of a bruit. A bruit is a __________ sound that is heard best with the __________ of the stethoscope. a. Low gurgling; diaphragm b. Loud, whooshing, blowing; bell c. Soft, whooshing, pulsatile; bell d. High-pitched tinkling; diaphragm C If the thyroid gland is enlarged, then the nurse should auscultate it for the presence of a bruit, which is a soft, pulsatile, whooshing, blowing sound heard best with the bell of the stethoscope. 24. The nurse notices that an infant has a large, soft lump on the side of his head and that his mother is very concerned. She tells the nurse that she noticed the lump approximately 8 hours after her baby’s birth and that it seems to be getting bigger. One possible explanation for this is: a. Hydrocephalus. b. Craniosynostosis. c. Cephalhematoma. d. Caput succedaneum. C A cephalhematoma is a subperiosteal hemorrhage that is the result of birth trauma. It is soft, fluctuant, and well defined over one cranial bone. It appears several hours after birth and gradually increases in size. 25. A mother brings in her newborn infant for an assessment and tells the nurse that she has noticed that whenever her newborn’s head is turned to the right side, she straightens out the arm and leg on the same side and flexes the opposite arm and leg. After observing this on examination, the nurse tells her that this reflex is: a. Abnormal and is called the atonic neck reflex. b. Normal and should disappear by the first year of life. c. Normal and is called the tonic neck reflex, which should disappear between 3 and 4 months of age. d. Abnormal. The baby should be flexing the arm and leg on the right side of his body when the head is turned to the right. C By 2 weeks, the infant shows the tonic neck reflex when supine and the head is turned to one side (extension of same arm and leg, flexion of opposite arm and leg). The tonic neck reflex disappears between 3 and 4 months of age. 26. During an admission assessment, the nurse notices that a male patient has an enlarged and rather thick skull. The nurse suspects acromegaly and would further assess for: a. Exophthalmos. b. Bowed long bones. c. Coarse facial features. d. Acorn-shaped cranium. C Acromegaly is excessive secretion of growth hormone that creates an enlarged skull and thickened cranial bones. Patients will have elongated heads, massive faces, prominent noses and lower jaws, heavy eyebrow ridges, and coarse facial features. Exophthalmos is associated with hyperthyroidism. Bowed long bones and an acorn-shaped cranium result from Paget disease. 27. When examining children affected with Down syndrome (trisomy 21), the nurse looks for the possible presence of: a. Ear dysplasia. b. Long, thin neck. c. Protruding thin tongue. d. Narrow and raised nasal bridge. A With the chromosomal aberration trisomy 21, also known as Down syndrome, head and face characteristics may include upslanting eyes with inner epicanthal folds, a flat nasal bridge, a small broad flat nose, a protruding thick tongue, ear dysplasia, a short broad neck with webbing, and small hands with a single palmar crease. 28. A patient visits the clinic because he has recently noticed that the left side of his mouth is paralyzed. He states that he cannot raise his eyebrow or whistle. The nurse suspects that he has: a. Cushing syndrome. b. Parkinson disease. c. Bell palsy. d. Experienced a cerebrovascular accident (CVA) or stroke. D With an upper motor neuron lesion, as with a CVA, the patient will have paralysis of lower facial muscles, but the upper half of the face will not be affected owing to the intact nerve from the unaffected hemisphere. The person is still able to wrinkle the forehead and close the eyes. (See Table 13-4, Abnormal Facial Appearances with Chronic Illnesses, for descriptions of the other responses.) 29. A woman comes to the clinic and states, “I’ve been sick for so long! My eyes have gotten so puffy, and my eyebrows and hair have become coarse and dry.” The nurse will assess for other signs and symptoms of: a. Cachexia. b. Parkinson syndrome. c. Myxedema. d. Scleroderma. C Myxedema (hypothyroidism) is a deficiency of thyroid hormone that, when severe, causes a nonpitting edema or myxedema. The patient will have a puffy edematous face, especially around the eyes (periorbital edema); coarse facial features; dry skin; and dry, coarse hair and eyebrows. (See Table 13-4, Abnormal Facial Appearances with Chronic Illnesses, for descriptions of the other responses.) 30. During an examination of a female patient, the nurse notes lymphadenopathy and suspects an acute infection. Acutely infected lymph nodes would be: a. Clumped. b. Unilateral. c. Firm but freely movable. d. Firm and nontender. C Acutely infected lymph nodes are bilateral, enlarged, warm, tender, and firm but freely movable. Unilaterally enlarged nodes that are firm and nontender may indicate cancer. 31. The physician reports that a patient with a neck tumor has a tracheal shift. The nurse is aware that this means that the patient’s trachea is: a. Pulled to the affected side. b. Pushed to the unaffected side. c. Pulled downward. d. Pulled downward in a rhythmic pattern. B The trachea is pushed to the unaffected side with an aortic aneurysm, a tumor, unilateral thyroid lobe enlargement, or a pneumothorax. The trachea is pulled to the affected side with large atelectasis, pleural adhesions, or fibrosis. Tracheal tug is a rhythmic downward pull that is synchronous with systole and occurs with aortic arch aneurysm. 32. During an assessment of an infant, the nurse notes that the fontanels are depressed and sunken. The nurse suspects which condition? a. Rickets b. Dehydration c. Mental retardation d. Increased intracranial pressure B Depressed and sunken fontanels occur with dehydration or malnutrition. Mental retardation and rickets have no effect on the fontanels. Increased intracranial pressure would cause tense or bulging and possibly pulsating fontanels. 33. The nurse is performing an assessment on a 7-year-old child who has symptoms of chronic watery eyes, sneezing, and clear nasal drainage. The nurse notices the presence of a transverse line across the bridge of the nose, dark blue shadows below the eyes, and a double crease on the lower eyelids. These findings are characteristic of: a. Allergies. b. Sinus infection. c. Nasal congestion. d. Upper respiratory infection. A Chronic allergies often develop chronic facial characteristics and include blue shadows below the eyes, a double or single crease on the lower eyelids, open-mouth breathing, and a transverse line on the nose. 34. While performing a well-child assessment on a 5 year old, the nurse notes the presence of palpable, bilateral, cervical, and inguinal lymph nodes. They are approximately 0.5 cm in size, round, mobile, and nontender. The nurse suspects that this child: a. Has chronic allergies. b. May have an infection. c. Is exhibiting a normal finding for a well child of this age. d. Should be referred for additional evaluation. C Palpable lymph nodes are normal in children until puberty when the lymphoid tissue begins to atrophy. Lymph nodes may be up to 1 cm in size in the cervical and inguinal areas but are discrete, movable, and nontender. 35. The nurse has just completed a lymph node assessment on a 60-year-old healthy female patient. The nurse knows that most lymph nodes in healthy adults are normally: a. Shotty. b. Nonpalpable. c. Large, firm, and fixed to the tissue. d. Rubbery, discrete, and mobile. B Most lymph nodes are nonpalpable in adults. The palpability of lymph nodes decreases with age. Normal nodes feel movable, discrete, soft, and nontender. 36. During an examination of a patient in her third trimester of pregnancy, the nurse notices that the patient’s thyroid gland is slightly enlarged. No enlargement had been previously noticed. The nurse suspects that the patient: a. Has an iodine deficiency. b. Is exhibiting early signs of goiter. c. Is exhibiting a normal enlargement of the thyroid gland during pregnancy. d. Needs further testing for possible thyroid cancer. C The thyroid gland enlarges slightly during pregnancy because of hyperplasia of the tissue and increased vascularity. 37. During an examination, the nurse knows that the best way to palpate the lymph nodes in the neck is described by which statement? a. Using gentle pressure, palpate with both hands to compare the two sides. b. Using strong pressure, palpate with both hands to compare the two sides. c. Gently pinch each node between one’s thumb and forefinger, and then move down the neck muscle. d. Using the index and middle fingers, gently palpate by applying pressure in a rotating pattern. A Using gentle pressure is recommended because strong pressure can push the nodes into the neck muscles. Palpating with both hands to compare the two sides symmetrically is usually most efficient. 38. During a well-baby checkup, a mother is concerned because her 2-month-old infant cannot hold her head up when she is pulled to a sitting position. Which response by the nurse is appropriate? a. “Head control is usually achieved by 4 months of age.” b. “You shouldn’t be trying to pull your baby up like that until she is older.” c. “Head control should be achieved by this time.” d. “This inability indicates possible nerve damage to the neck muscles.” A Head control is achieved by 4 months when the baby can hold the head erect and steady when pulled to a vertical position. The other responses are not appropriate. 39. During an examination of a 3-year-old child, the nurse notices a bruit over the left temporal area. The nurse should: a. Continue the examination because a bruit is a normal finding for this age. b. Check for the bruit again in 1 hour. c. Notify the parents that a bruit has been detected in their child. d. Stop the examination, and notify the physician. A Bruits are common in the skull in children under 4 or 5 years of age and in children with anemia. They are systolic or continuous and are heard over the temporal area. 40. During an examination, the nurse finds that a patient’s left temporal artery is tortuous and feels hardened and tender, compared with the right temporal artery. The nurse suspects which condition? a. Crepitation b. Mastoiditis c. Temporal arteritis d. Bell palsy C With temporal arteritis, the artery appears more tortuous and feels hardened and tender. These assessment findings are not consistent with the other responses. MULTIPLE RESPONSE 1. The nurse is assessing a 1-month-old infant at his well-baby checkup. Which assessment findings are appropriate for this age? Select all that apply. a. Head circumference equal to chest circumference b. Head circumference greater than chest circumference c. Head circumference less than chest circumference d. Fontanels firm and slightly concave e. Absent tonic neck reflex f. Nonpalpable cervical lymph nodes B, D, F An infant’s head circumference is larger than the chest circumference. At age 2 years, both measurements are the same. During childhood, the chest circumference grows to exceed the head circumference by 5 to 7 cm. The fontanels should feel firm and slightly concave in the infant, and they should close by age 9 months. The tonic neck reflex is present until between 3 and 4 months of age, and cervical lymph nodes are normally nonpalpable in an infant. CHAPER 16 1. The primary purpose of the ciliated mucous membrane in the nose is to: a. Warm the inhaled air. b. Filter out dust and bacteria. c. Filter coarse particles from inhaled air. d. Facilitate the movement of air through the nares. B The nasal hairs filter the coarsest matter from inhaled air, whereas the mucous blanket filters out dust and bacteria. The rich blood supply of the nasal mucosa warms the inhaled air. 2. The projections in the nasal cavity that increase the surface area are called the: a. Meatus. b. Septum. c. Turbinates. d. Kiesselbach plexus. C The lateral walls of each nasal cavity contain three parallel bony projections: the superior, middle, and inferior turbinates. These increase the surface area, making more blood vessels and mucous membrane available to warm, humidify, and filter the inhaled air. 3. The nurse is reviewing the development of the newborn infant. Regarding the sinuses, which statement is true in relation to a newborn infant? a. Sphenoid sinuses are full size at birth. b. Maxillary sinuses reach full size after puberty. c. Frontal sinuses are fairly well developed at birth. d. Maxillary and ethmoid sinuses are the only sinuses present at birth. D Only the maxillary and ethmoid sinuses are present at birth. The sphenoid sinuses are minute at birth and develop after puberty. The frontal sinuses are absent at birth, are fairly well developed at age 7 to 8 years, and reach full size after puberty. 4. The tissue that connects the tongue to the floor of the mouth is the: a. Uvula. b. Palate. c. Papillae. d. Frenulum. D The frenulum is a midline fold of tissue that connects the tongue to the floor of the mouth. The uvula is the free projection hanging down from the middle of the soft palate. The palate is the arching roof of the mouth. Papillae are the rough, bumpy elevations on the tongue’s dorsal surface. 5. The salivary gland that is the largest and located in the cheek in front of the ear is the _________ gland. a. Parotid b. Stensen’s c. Sublingual d. Submandibular A The mouth contains three pairs of salivary glands. The largest, the parotid gland, lies within the cheeks in front of the ear extending from the zygomatic arch down to the angle of the jaw. The Stensen’s duct (not gland) drains the parotid gland onto the buccal mucosa opposite the second molar. The sublingual gland is located within the floor of the mouth under the tongue. The submandibular gland lies beneath the mandible at the angle of the jaw. 6. In assessing the tonsils of a 30 year old, the nurse notices that they are involuted, granular in appearance, and appear to have deep crypts. What is correct response to these findings? a. Refer the patient to a throat specialist. b. No response is needed; this appearance is normal for the tonsils. c. Continue with the assessment, looking for any other abnormal findings. d. Obtain a throat culture on the patient for possible streptococcal (strep) infection. B The tonsils are the same color as the surrounding mucous membrane, although they look more granular and their surface shows deep crypts. Tonsillar tissue enlarges during childhood until puberty and then involutes. 7. The nurse is obtaining a health history on a 3-month-old infant. During the interview, the mother states, “I think she is getting her first tooth because she has started drooling a lot.” The nurse’s best response would be: a. “You’re right, drooling is usually a sign of the first tooth.” b. “It would be unusual for a 3 month old to be getting her first tooth.” c. “This could be the sign of a problem with the salivary glands.” d. “She is just starting to salivate and hasn’t learned to swallow the saliva.” D In the infant, salivation starts at 3 months. The baby will drool for a few months before learning to swallow the saliva. This drooling does not herald the eruption of the first tooth, although many parents think it does. 8. The nurse is assessing an 80-year-old patient. Which of these findings would be expected for this patient? a. Hypertrophy of the gums b. Increased production of saliva c. Decreased ability to identify odors d. Finer and less prominent nasal hair C The sense of smell may be reduced because of a decrease in the number of olfactory nerve fibers. Nasal hairs grow coarser and stiffer with aging. The gums may recede with aging, not hypertrophy, and saliva production decreases. 9. The nurse is performing an oral assessment on a 40-year-old Black patient and notices the presence of a 1 cm, nontender, grayish-white lesion on the left buccal mucosa. Which one of these statements is true? This lesion is: a. Leukoedema and is common in dark-pigmented persons. b. The result of hyperpigmentation and is normal. c. Torus palatinus and would normally be found only in smokers. d. Indicative of cancer and should be immediately tested. A Leukoedema, a grayish-white benign lesion occurring on the buccal mucosa, is most often observed in Blacks. 10. While obtaining a health history, a patient tells the nurse that he has frequent nosebleeds and asks the best way to get them to stop. What would be the nurse’s best response? a. “While sitting up, place a cold compress over your nose.” b. “Sit up with your head tilted forward and pinch your nose.” c. “Just allow the bleeding to stop on its own, but don’t blow your nose.” d. “Lie on your back with your head tilted back and pinch your nose.” B With a nosebleed, the person should sit up with the head tilted forward and pinch the nose between the thumb and forefinger for 5 to 15 minutes. 11. A 92-year-old patient has had a stroke. The right side of his face is drooping. The nurse might also suspect which of these assessment findings? a. Epistaxis b. Rhinorrhea c. Dysphagia d. Xerostomia C Dysphagia is difficulty with swallowing and may occur with a variety of disorders, including stroke and other neurologic diseases. Rhinorrhea is a runny nose, epistaxis is a bloody nose, and xerostomia is a dry mouth. 12. While obtaining a health history from the mother of a 1-year-old child, the nurse notices that the baby has had a bottle in his mouth the entire time. The mother states, “It makes a great pacifier.” The best response by the nurse would be: a. “You’re right. Bottles make very good pacifiers.” b. “Using a bottle as a pacifier is better for the teeth than thumb-sucking.” c. “It’s okay to use a bottle as long as it contains milk and not juice.” d. “Prolonged use of a bottle can increase the risk for tooth decay and ear infections.” D Prolonged bottle use during the day or when going to sleep places the infant at risk for tooth decay and middle ear infections. 13. A 72-year-old patient has a history of hypertension and chronic lung disease. An important question for the nurse to include in the health history would be: a. “Do you use a fluoride supplement?” b. “Have you had tonsillitis in the last year?” c. “At what age did you get your first tooth?” d. “Have you noticed any dryness in your mouth?” D Xerostomia (dry mouth) is a side effect of many drugs taken by older people, including antidepressants, anticholinergics, antispasmodics, antihypertensives, antipsychotics, and bronchodilators. 14. The nurse is using an otoscope to assess the nasal cavity. Which of these techniques is correct? a. Inserting the speculum at least 3 cm into the vestibule b. Avoiding touching the nasal septum with the speculum c. Gently displacing the nose to the side that is being examined d. Keeping the speculum tip medial to avoid touching the floor of the nares B The correct technique for using an otoscope is to insert the apparatus into the nasal vestibule, avoiding pressure on the sensitive nasal septum. The tip of the nose should be lifted up before inserting the speculum. 15. The nurse is performing an assessment on a 21-year-old patient and notices that his nasal mucosa appears pale, gray, and swollen. What would be the most appropriate question to ask the patient? a. “Are you aware of having any allergies?” b. “Do you have an elevated temperature?” c. “Have you had any symptoms of a cold?” d. “Have you been having frequent nosebleeds?” A With chronic allergies, the mucosa looks swollen, boggy, pale, and gray. Elevated body temperature, colds, and nosebleeds do not cause these mucosal changes. 16. The nurse is palpating the sinus areas. If the findings are normal, then the patient should report which sensation? a. No sensation b. Firm pressure c. Pain during palpation d. Pain sensation behind eyes B The person should feel firm pressure but no pain. Sinus areas are tender to palpation in persons with chronic allergies or an acute infection (sinusitis). 17. During an oral assessment of a 30-year-old Black patient, the nurse notices bluish lips and a dark line along the gingival margin. What action would the nurse perform in response to this finding? a. Check the patient’s hemoglobin for anemia. b. Assess for other signs of insufficient oxygen supply. c. Proceed with the assessment, knowing that this appearance is a normal finding. d. Ask if he has been exposed to an excessive amount of carbon monoxide. C Some Blacks may have bluish lips and a dark line on the gingival margin; this appearance is a normal finding. 18. During an assessment of a 20-year-old patient with a 3-day history of nausea and vomiting, the nurse notices dry mucosa and deep vertical fissures in the tongue. These findings are reflective of: a. Dehydration. b. Irritation by gastric juices. c. A normal oral assessment. d. Side effects from nausea medication. A Dry mouth occurs with dehydration or fever. The tongue has deep vertical fissures. 19. A 32-year-old woman is at the clinic for “little white bumps in my mouth.” During the assessment, the nurse notes that she has a 0.5 cm white, nontender papule under her tongue and one on the mucosa of her right cheek. What would the nurse tell the patient? a. “These spots indicate an infection such as strep throat.” b. “These bumps could be indicative of a serious lesion, so I will refer you to a specialist.” c. “This condition is called leukoplakia and can be caused by chronic irritation such as with smoking.” d. “These bumps are Fordyce granules, which are sebaceous cysts and are not a serious condition.” D Fordyce granules are small, isolated white or yellow papules on the mucosa of the cheek, tongue, and lips. These little sebaceous cysts are painless and are not significant. Chalky, white raised patches would indicate leukoplakia. In strep throat, the examiner would see tonsils that are bright red, swollen, and may have exudates or white spots. 20. A 10 year old is at the clinic for “a sore throat that has lasted 6 days.” Which of these findings would be consistent with an acute infection? a. Tonsils 1+/1-4+ and pink; the same color as the oral mucosa b. Tonsils 2+/1-4+ with small plugs of white debris c. Tonsils 3+/1-4+ with large white spots d. Tonsils 3+/1-4+ with pale coloring C With an acute infection, tonsils are bright red and swollen and may have exudate or large white spots. Tonsils are enlarged to 2+, 3+, or 4+ with an acute infection. 21. Immediately after birth, the nurse is unable to suction the nares of a newborn. An attempt is made to pass a catheter through both nasal cavities with no success. What should the nurse do next? a. Attempt to suction again with a bulb syringe. b. Wait a few minutes, and try again once the infant stops crying. c. Recognize that this situation requires immediate intervention. d. Contact the physician to schedule an appointment for the infant at his or her next hospital visit. C Determining the patency of the nares in the immediate newborn period is essential because most newborns are obligate nose breathers. Nares blocked with amniotic fluid are gently suctioned with a bulb syringe. If obstruction is suspected, then a small lumen (5 to 10 Fr) catheter is passed down each naris to confirm patency. The inability to pass a catheter through the nasal cavity indicates choanal atresia, which requires immediate intervention. 22. The nurse notices that the mother of a 2-year-old boy brings him into the clinic quite frequently for various injuries and suspects there may be some child abuse involved. During an inspection of his mouth, the nurse should look for: a. Swollen, red tonsils. b. Ulcerations on the hard palate. c. Bruising on the buccal mucosa or gums. d. Small yellow papules along the hard palate. C The nurse should notice any bruising or laceration on the buccal mucosa or gums of an infant or young child. Trauma may indicate child abuse from a forced feeding of a bottle or spoon. 23. The nurse is assessing a 3 year old for “drainage from the nose.” On assessment, a purulent drainage that has a very foul odor is noted from the left naris and no drainage is observed from the right naris. The child is afebrile with no other symptoms. What should the nurse do next? a. Refer to the physician for an antibiotic order. b. Have the mother bring the child back in 1 week. c. Perform an otoscopic examination of the left nares. d. Tell the mother that this drainage is normal for a child of this age. C Children are prone to put an object up the nose, producing unilateral purulent drainage with a foul odor. Because some risk for aspiration exists, removal should be prompt. 24. During an assessment of a 26 year old at the clinic for “a spot on my lip I think is cancer,” the nurse notices a group of clear vesicles with an erythematous base around them located at the lip-skin border. The patient mentions that she just returned from Hawaii. What would be the most appropriate response by the nurse? a. Tell the patient she needs to see a skin specialist. b. Discuss the benefits of having a biopsy performed on any unusual lesion. c. Tell the patient that these vesicles are indicative of herpes simplex I or cold sores and that they will heal in 4 to 10 days. d. Tell the patient that these vesicles are most likely the result of a riboflavin deficiency and discuss nutrition. C Cold sores are groups of clear vesicles with a surrounding erythematous base. These evolve into pustules or crusts and heal in 4 to 10 days. The most likely site is the lip-skin junction. Infection often recurs in the same site. Recurrent herpes infections may be precipitated by sunlight, fever, colds, or allergy. 25. While performing an assessment of the mouth, the nurse notices that the patient has a 1-cm ulceration that is crusted with an elevated border and located on the outer third of the lower lip. What other information would be most important for the nurse to assess? a. Nutritional status b. When the patient first noticed the lesion c. Whether the patient has had a recent cold d. Whether the patient has had any recent exposure to sick animals B With carcinoma, the initial lesion is round and indurated, but then it becomes crusted and ulcerated with an elevated border. Most cancers occur between the outer and middle thirds of the lip. Any lesion that is still unhealed after 2 weeks should be referred. 26. A pregnant woman states that she is concerned about her gums because she has noticed they are swollen and have started bleeding. What would be an appropriate response by the nurse? a. “Your condition is probably due to a vitamin C deficiency.” b. “I’m not sure what causes swollen and bleeding gums, but let me know if it’s not better in a few weeks.” c. “You need to make an appointment with your dentist as soon as possible to have this checked.” d. “Swollen and bleeding gums can be caused by the change in hormonal balance in your system during pregnancy.” D Gum margins are red and swollen and easily bleed with gingivitis. A changing hormonal balance may cause this condition to occur in pregnancy and puberty. 27. A 40-year-old patient who has just finished chemotherapy for breast cancer tells the nurse that she is concerned about her mouth. During the assessment the nurse finds areas of buccal mucosa that are raw and red with some bleeding, as well as other areas that have a white, cheesy coating. The nurse recognizes that this abnormality is: a. Aphthous ulcers. b. Candidiasis. c. Leukoplakia. d. Koplik spots. B Candidiasis is a white, cheesy, curdlike patch on the buccal mucosa and tongue. It scrapes off, leaving a raw, red surface that easily bleeds. It also occurs after the use of antibiotics or corticosteroids and in persons who are immunosuppressed. (See Table 16-4 for descriptions of the other lesions.) 28. The nurse is assessing a patient in the hospital who has received numerous antibiotics and notices that his tongue appears to be black and hairy. In response to his concern, what would the nurse say? a. “We will need to get a biopsy to determine the cause.” b. “This is an overgrowth of hair and will go away in a few days.” c. “Black, hairy tongue is a fungal infection caused by all the antibiotics you have received.” d. “This is probably caused by the same bacteria you had in your lungs.” C A black, hairy tongue is not really hair but the elongation of filiform papillae and painless overgrowth of mycelial threads of fungus infection on the tongue. It occurs after the use of antibiotics, which inhibit normal bacteria and allow a proliferation of fungus. 29. The nurse is assessing a patient with a history of intravenous drug abuse. In assessing his mouth, the nurse notices a dark red confluent macule on the hard palate. This could be an early sign of: a. Acquired immunodeficiency syndrome (AIDS). b. Measles. c. Leukemia. d. Carcinoma. A Oral Kaposi’s sarcoma is a bruiselike, dark red or violet, confluent macule that usually occurs on the hard palate. It may appear on the soft palate or gingival margin. Oral lesions may be among the earliest lesions to develop with AIDS. 30. A mother brings her 4-month-old infant to the clinic with concerns regarding a small pad in the middle of the upper lip that has been there since 1 month of age. The infant has no health problems. On physical examination, the nurse notices a 0.5-cm, fleshy, elevated area in the middle of the upper lip. No evidence of inflammation or drainage is observed. What would the nurse tell this mother? a. “This area of irritation is caused from teething and is nothing to worry about.” b. “This finding is abnormal and should be evaluated by another health care provider.” c. “This area of irritation is the result of chronic drooling and should resolve within the next month or two.” d. “This elevated area is a sucking tubercle caused from the friction of breastfeeding or bottle-feeding and is normal.” D A normal finding in infants is the sucking tubercle, a small pad in the middle of the upper lip from the friction of breastfeeding or bottle-feeding. This condition is not caused by irritation, teething, or excessive drooling, and evaluation by another health care provider is not warranted. 31. A mother is concerned because her 18-month-old toddler has 12 teeth. She is wondering if this is normal for a child of this age. The nurse’s best response would be: a. “How many teeth did you have at this age?” b. “All 20 deciduous teeth are expected to erupt by age 4 years.” c. “This is a normal number of teeth for an 18 month old.” d. “Normally, by age 2 years, 16 deciduous teeth are expected.” C The guidelines for the number of teeth for children younger than 2 years old are as follows: the child’s age in months minus the number 6 should be equal to the expected number of deciduous teeth. Normally, all 20 teeth are in by 2 years old. In this instance, the child is 18 months old, minus 6, equals 12 deciduous teeth expected. 32. When examining the mouth of an older patient, the nurse recognizes which finding is due to the aging process? a. Teeth appearing shorter b. Tongue that looks smoother in appearance c. Buccal mucosa that is beefy red in appearance d. Small, painless lump on the dorsum of the tongue B In the aging adult, the tongue looks smoother because of papillary atrophy. The teeth are slightly yellowed and appear longer because of the recession of gingival margins. 33. When examining the nares of a 45-year-old patient who has complaints of rhinorrhea, itching of the nose and eyes, and sneezing, the nurse notices the following: pale turbinates, swelling of the turbinates, and clear rhinorrhea. Which of these conditions is most likely the cause? a. Nasal polyps b. Acute sinusitis c. Allergic rhinitis d. Acute rhinitis C Rhinorrhea, itching of the nose and eyes, and sneezing are present with allergic rhinitis. On physical examination, serous edema is noted, and the turbinates usually appear pale with a smooth, glistening surface. (See Table 16-1 for descriptions of the other conditions.) 34. When assessing the tongue of an adult, the nurse knows that an abnormal finding would be: a. Smooth glossy dorsal surface. b. Thin white coating over the tongue. c. Raised papillae on the dorsal surface. d. Visible venous patterns on the ventral surface. A The dorsal surface of the tongue is normally roughened from papillae. A thin white coating may be present. The ventral surface may show veins. Smooth, glossy areas may indicate atrophic glossitis (see Table 16-5). 35. The nurse is performing an assessment. Which of these findings would cause the greatest concern? a. Painful vesicle inside the cheek for 2 days b. Presence of moist, nontenderStensen’s ducts c. Stippled gingival margins that snugly adhere to the teeth d. Ulceration on the side of the tongue with rolled edges D Ulceration on the side or base of the tongue or under the tongue raises the suspicion of cancer and must be investigated. The risk of early metastasis is present because of rich lymphatic drainage. The vesicle may be an aphthous ulcer, which is painful but not dangerous. The other responses are normal findings. 36. A patient has been diagnosed with strep throat. The nurse is aware that without treatment, which complication may occur? a. Rubella b. Leukoplakia c. Rheumatic fever d. Scarlet fever C Untreated strep throat may lead to rheumatic fever. When performing a health history, the patient should be asked whether his or her sore throat has been documented as streptococcal. 37. During a checkup, a 22-year-old woman tells the nurse that she uses an over-the-counter nasal spray because of her allergies. She also states that it does not work as well as it used to when she first started using it. The best response by the nurse would be: a. “You should never use over-the-counter nasal sprays because of the risk of addiction.” b. “You should try switching to another brand of medication to prevent this problem.” c. “Continuing to use this spray is important to keep your allergies under control.” d. “Using these nasal medications irritates the lining of the nose and may cause rebound swelling.” D The misuse of over-the-counter nasal medications irritates the mucosa, causing rebound swelling, which is a common problem. 38. During an oral examination of a 4-year-old Native-American child, the nurse notices that her uvula is partially split. Which of these statements is accurate? a. This condition is a cleft palate and is common in Native Americans. b. A bifid uvula may occur in some Native-American groups. c. This condition is due to an injury and should be reported to the authorities. d. A bifid uvula is palatinus, which frequently occurs in Native Americans. B Bifid uvula, a condition in which the uvula is split either completely or partially, occurs in some Native-American groups. 39. A patient comes into the clinic complaining of facial pain, fever, and malaise. On examination, the nurse notes swollen turbinates and purulent discharge from the nose. The patient also complains of a dull, throbbing pain in his cheeks and teeth on the right side and pain when the nurse palpates the areas. The nurse recognizes that this patient has: a. Posterior epistaxis. b. Frontal sinusitis. c. Maxillary sinusitis. d. Nasal polyps. C Signs of maxillary sinusitis include facial pain after upper respiratory infection, red swollen nasal mucosa, swollen turbinates, and purulent discharge. The person also has fever, chills, and malaise. With maxillary sinusitis, dull throbbing pain occurs in the cheeks and teeth on the same side, and pain with palpation is present. With frontal sinusitis, pain is above the supraorbital ridge. 40. A woman who is in the second trimester of pregnancy mentions that she has had “more nosebleeds than ever” since she became pregnant. The nurse recognizes that this is a result of: a. A problem with the patient’s coagulation system. b. Increased vascularity in the upper respiratory tract as a result of the pregnancy. c. Increased susceptibility to colds and nasal irritation. d. Inappropriate use of nasal sprays. B Nasal stuffiness and epistaxis may occur during pregnancy as a result of increased vascularity in the upper respiratory tract. MULTIPLE RESPONSE 1. The nurse is teaching a health class to high-school boys. When discussing the topic of using smokeless tobacco (SLT), which of these statements are accurate? Select all that apply. a. One pinch of SLT in the mouth for 30 minutes delivers the equivalent of one cigarette. b. Using SLT has been associated with a greater risk of oral cancer than smoking. c. Pain is an early sign of oral cancer. d. Pain is rarely an early sign of oral cancer. e. Tooth decay is another risk of SLT because of the use of sugar as a sweetener. f. SLT is considered a healthy alternative to smoking. B, D, E One pinch of SLT in the mouth for 30 minutes delivers the equivalent of three cigarettes. Pain is rarely an early sign of oral cancer. Many brands of SLT are sweetened with sugars, which promotes tooth decay. SLT is not considered a healthy alternative to smoking, and the use of SLT has been associated with a greater risk of oral cancer than smoking. 2. During an assessment, a patient mentions that “I just can’t smell like I used to. I can barely smell the roses in my garden. Why is that?” For which possible causes of changes in the sense of smell will the nurse assess? Select all that apply. a. Chronic alcohol use b. Cigarette smoking c. Frequent episodes of strep throat d. Chronic allergies e. Aging f. Herpes simplex virus I B, D, E Sen The sense of smell diminishes with cigarette smoking, chronic allergies, and aging. Chronic alcohol use, a history of strep throat, and herpes simplex virus I are not associated with changes in the sense of smell. [Show More]

Last updated: 1 year ago

Preview 1 out of 123 pages

Add to cart

Instant download

document-preview

Buy this document to get the full access instantly

Instant Download Access after purchase

Add to cart

Instant download

Reviews( 0 )

$18.00

Add to cart

Instant download

Can't find what you want? Try our AI powered Search

OR

REQUEST DOCUMENT
44
0

Document information


Connected school, study & course


About the document


Uploaded On

Feb 12, 2021

Number of pages

123

Written in

Seller


seller-icon
securegrades

Member since 4 years

117 Documents Sold


Additional information

This document has been written for:

Uploaded

Feb 12, 2021

Downloads

 0

Views

 44

Document Keyword Tags

Recommended For You

Get more on EXAM »

$18.00
What is Browsegrades

In Browsegrades, a student can earn by offering help to other student. Students can help other students with materials by upploading their notes and earn money.

We are here to help

We're available through e-mail, Twitter, Facebook, and live chat.
 FAQ
 Questions? Leave a message!

Follow us on
 Twitter

Copyright © Browsegrades · High quality services·